Sie sind auf Seite 1von 65

TABLE OF CONTENTS

1. FEDERAL JUDICIAL POWER..PG. 2


JUDICIAL REVIEW PG. 2 LIMITATIONS ON JUDICIAL POWER PG. 3 JUSTICIABILITY LIMITS PG. 5 CONSTITUTIONAL ANALYSIS: ACTS OF CONGRESS PG. 10 COMMERCE POWER PG. 11 TAXING & SPENDING POWER PG. 23 POWER UNDER CIVIL RIGHTS AMENDMENTS PG. 24 INHERENT PRESIDENTIAL POWER PG. 30 NON-DELEGATION DOCTRINE PG. 32 LEGISLATIVE VETO PG. 32 CHECKING ADMINISTRATIVE AGENCIES PG. 33 PRESIDENTIAL WAR POWERS PG. 35 SUSPENSION CLAUSE PG. 36 MILITARY TRIBUNALS PG. 37 PRESIDENTIAL IMMUNITY PG. 37 IMPEACHMENT PG. 38 PREEMPTION PG. 39 DORMANT COMMERCE CLAUSE PG. 41 APPLICATION OF THE BILL OF RIGHTS TO THE STATES PG. 43

2. FEDERAL LEGISLATIVE POWER..PG. 10

3. FEDERAL EXECUTIVE POWER.PG. 30

4. LIMITS ON STATE REGULATORY & TAXING POWER.PG. 39 5. CONSTITUTIONAL PROTECTION OF CIVIL RIGHTS & LIBERTIES PG. 43 6. 7.
INCORPORATION OF THE BILL OF RIGHTS INTO DUE PROCESS CLAUSE PG. 45 ECONOMIC LIBERTIES.PG. 48

LOCHNER ERA PG. 48

POST-LOCHNER: 1937-PRESENT PG. 49 FUNDAMENTAL RIGHTS UNDER DUE PROCESS & EQUAL PROTECTION...PG. 51

FUNDAMENTAL RIGHT ANALYSIS PG. 51 CONSTITUTIONAL PROTECTION FOR FAMILY AUTONOMY PG. 51 CONTRACEPTION CASES PG. 54 ABORTION CASES PG. 55 ABORTION RESTRICTIONS PG. 59 CONSTITUTIONAL PROTECTIONS FOR MEDICAL DECISIONS PG. 66 CONSTITUTIONAL PROTECTIONS FOR SEXUAL ORIENTATION & ACTIVITY PG. 53 SEPARATION OF POWERS PG. 64 LIST OF CONSTITUTIONAL CLAUSES PG. 64

8. APPENDIX..PG. 64

THE FEDERAL JUDICIAL POWER ARTICLE III


Reason for a constitution and not statutes: Provides a structure for government and delegates powers. Much harder to amend a constitution. Certain issues should put out of reach from the democratic majorities to protect the minorities. In times of emergency, it protects from drastic changes in the structure of the government. Protects democracy. 1781 Articles of Confederation Too loose, more centralized federal government need. Formation of the Constitution disregarded AoCs requirement of unanimous ratification by the 13 states, only 9 states ratified. American government was formed illegally. 1789 Bill of Rights (1st 10 Amendments) Ratified in 1791. Constitution is short because it uses very broad, vague language. What does the Constitution do? Provides a structure for government and delegates powers. Protecting measures of constitutional rights. o Art. I - Section 9 & 10 o Bill of Rights, Amendments o Amendment 10 Powers not delegated by the Constitution are reserved for the States

JUDICIAL REVIEW
Marbury v. Madison (1803) pg. 2 Facts: Marbury was appointed a justice of the peace by the defeated incumbent Federalist President, John Adams, at the end of his administration. The Federalist-controlled Senate confirmed the appointments of Adams last-minute nominees. The formal commissions had not been delivered when Jefferson, the new Republican President, took office a few days later. Jefferson refused to deliver the commissions of the justices appointed by Adams. Marbury sought a writ of mandamus to compel Madison (Jeffersons secretary of state) to deliver the commissions. Holding: A law repugnant to the Constitution is void. Courts are bound by the Constitution they do not have discretion whether or not to follow it. Courts say what the law is (establishing judicial review). Reasoning: Marshalls argument follows six steps. The last three are the most important. 1. Did Marbury have a vested legal right to his commission? Yes; it was signed by the Sec. of State. 2. If it was a vested right, is there a legal remedy for deprivation of the right? Yes; the duty of govt is to afford protection of the laws. Govt must provide a remedy for violation of vested property rights. Justification for this argument: essence/duty of govt (no textual basis) 3. Was this situation an exception from (2)? Exceptions to the principle of civil liberty are based on the nature and essence of the act involved. Is the act political/discretionary or a matter of duty? (Dealing w/individual rights duty; respecting the nation political) If political, no remedy. 4. Given that a remedy is appropriate, what remedy and from what court? Congress gave SCOTUS original jurisdiction to issue writs of mandamus in the Judiciary Act of 1789, but the Court asks if that provision of the Act is constitutional? NO. Justification: Art. III, 2 (though leaves out the crucial last part about Congress making exceptions/regulations) in all other 2

cases the court shall have appellate jurisdiction. Marshalls argument is not strong, but declares Act unconstitutional according to the texts discoverable meaning (law separate from politics). 5. Is a law at odds with the Constitution a valid law? The establishment of a Constitution is a great exertion, not done often, so its principles are deemed fundamental. This is a bright-line approach (no middle ground) that focuses on the fear of legislative omnipotence/tyranny. 6. If a statute made by a coequal branch contradicts the discoverable meaning of the Const, may the court apply this invalid law? NO. Focus on the definite meaning of the text. Discussion: This case establishing the concept of judicial review was steeped in ordinary politicshardly transcendent. Yet the language of the opinion suggests a belief that Constitutional law is transcendent. Marshall makes assumptions about judges as opposed to politicians (more qualified, less partisan) A distrust of the people- they ought to act rarely. Marbury appointed Justice of the Peace by President Adams. Appointment was signed sealed BUT not delivered when President Jefferson ordered James Madison to order CJ Marshall to stop delivering the notice of appointments. Marburys claim is constitutional (Article II Section 2 appointment of U.S. officers) and statutory (Judiciary Act of 1789 Writ of mandamus). SCOTUS finds that granting the writ of mandamus would violate the constitution because Supreme Court does not have original jurisdiction. Article III Section 2 Establishes only original jurisdiction in cases affecting ambassadors, public ministers and consuls, and those in which a state is a party. For all other cases, SCOTUS has appellate jurisdiction both as to law and fact. Article VI Clause 2 Supremacy Clause When statutes conflict with the Constitution, the constitution governs. Justify judicial review because the constitution is Law. (Pg. 6) Also says that if the legislature were able to change the Constitution (which restricts legislative power) then they could delegate as much power to themselves as possible. Power for Judicial review (of the Constitution) by SCOTUS is established by Marbury. Gives them the last word as to the interpretation of the Constitution. Judicial Review Checks and balances Impartial not influenced by political pressures Martin v. Hunters Lessee (1816) pg. 10 Dispute over the ownership of land. SCOTUS Rules federal treaty is controlling. Sends it back to Virginia COA. VCOA says SCOTUS has no authority to review state court decisions. SCOTUS replies the Constitution grants them the power to review state court decisions via Article 6 2 Supremacy clause.

LIMITATIONS ON JUDICIAL POWER


1. Interpretive Limits Originalism Interpretation of the Constitution should be confined to enforcing norms that are stated or clearly implicit in the written Constitution. (Intent of the Framers) Constitutional rights are enshrined with the scope they were understood to have when the people adopted them.

Non-originalism Courts should go beyond that set of references and enforce norms that cannot be discovered within the four corners of the document so that the Constitution does not remain virtually static and can meet the needs of society. District of Columbia v. Heller (2008) pg. 13 Amendment II A well-regulated militia, being necessary to the security of a free state, the right of the people, to keep and bear arms, shall not be infringed. Majority and Dissent - Purpose of the 2nd is to protect the people and the states from a potential tyrannical federal government. Issue is whether there is a link to a state militia. Majority says no, Dissent says yes. The right of the people, to keep and bear arms, shall not be infringed is the Operative clause. A well-regulated militia, being necessary to the security of a free state is a Prefatory clause. Constitutional rights are enshrined with the scope they were understood to have when the people adopted them. Arms are defined as they where in the 18th-century Dissent: (Breyer) Determination on whether the statute is unconstitutional requires us to focus on practicalities. Original purposes should be applied to modern-day circumstances. Deciding this case requires judicial judgment. Scalias Argument for Originalism If the constitution were not fixed law, but an invention to apply current societal values, why would the invitation of judicial review be addressed to the court and not the legislature? The legislature would seem a much more appropriate expositor of social values. Almost every originalist would adulterate it with the doctrine of stare decisis so that Marbury v. Madison would stand even if Professor Raoul Berger should demonstrate unassailably that it got the meaning of the constitution wrong. There would sometimes be cases where originalists would go against originalism. Ex. Ear-cropping and the 8th Amendment. *Stare Decisis applies to standing by your previous positions. Only a part of Precedence doctrine. 2. Congressional Limits Exceptions & Regulations Clause Separation of Powers Article III SEC 1 The judicial power of the US shall be vested in one Supreme Court and in such inferior Courts as the Congress may from time to time ordain and establish SEC 2 Exceptions & Regulations Clause In all other cases before mentioned, the Supreme Court shall have appellate jurisdiction both, as to law and fact, with such exceptions, and under such regulations as the Congress shall make. Congress cannot abolish inferior courts because of separation of powers. Ex Parte McCardle pg. 35 (1816) E&R clause limitations Imprisoned in 1868 for comments made about Reconstruction in the South after the Civil War. Files a writ of habeas corpus. Law enabling federal courts the power to grant habeas corpus relief enacted in 1867. Congress repeals that statute to prevent Reconstruction Act from being deemed unconstitutional. Question is whether the court has jurisdiction? NO. Appellate Jurisdiction is conferred by the Constitution with exceptions and under regulations Congress makes. McCardle Arguments: 4

Military Reconstruction Act provision providing military trials for civilians Unconstitutional. Prosecution violated 1st, 5th, & 6th Amendments. The 1st question is necessarily that of jurisdiction; for if the act of March 1868, takes away jurisdiction defined by the act of Feb. 1867, it is useless, if not improper, to enter into any discussion of other questions. If Congress can take away SCOTUSs appellate jurisdiction then it can regulate jurisdiction. Aftermath: Ex Parte Yerger Court upheld appellate jurisdiction to review habeas corpus decisions of lower federal courts under Judiciary Act of 1789 (review of Constitutional Claims). United States v. Klein (1871) pg. 37 A person who had property confiscated during the civil war could receive compensation if they could prove they had not assisted the enemy. The Supreme Court decided a Presidential pardon could prove such innocence. Congress passed opposing law declaring pardons inadmissible as evidence of innocence in a claim for the return of seized property, & that upon proof of such a pardon the jurisdiction of the court would cease to exist Court commentary acceptance of a pardon, without disclaimer is conclusive evidence of the pardoned acts, but are null & void as evidence of the rights conferred by it. Holding applies only in a situation where Congress directs the judiciary as to decision making under an existing law and does not apply when Congress adopts a New law. Robertson v. Seattle Audobon Society. Supreme Court rules the law infringes on the powers of the judicial branch and executive branch. Opinion is Confusing leaves the law murky. Difference between McCardle and Klein: Klein - Congress is trying to interfere with the Powers of the Executive Branch (President) and the Power of the Judicial Branchs (SCOTUS) ability to interpret the Presidents power. Overtime, more people believe jurisdiction stripping is a violation of Separation of Powers.

JUSTICIABILITY LIMITS PG. 40


Doctrines that limit the matters that can be heard in the federal courts. 1. Prohibition against advisory opinions 2. Standing 3. Ripeness 4. Mootness 5. Political question 1-4 arise from Article III 2 (Clause 1 Case & Controversy Clause). *Political Question falls under separation of powers NOT case & controversy clause but SCOTUS has addressed it as a part of C&C clause. Political Question arises under Separation of Powers. 1. Prohibition Against Advisory Opinions There must be an actual dispute between adverse litigants, and a substantial likelihood that a federal court decision in favor of a claimant will bring about some change or have some effect. Reasons: Limit the powers of the court Judicial economy When you have actual litigants they have actual interest, so they will make the best arguments for both sides. With advisory opinions you do not have this benefit. Making decisions without understanding the grievances of the people who will be affected by the law.

Interpretation: Clause distinguishes between lawsuits within and beyond the institutional competence of the federal judiciary, and extends the judicial power only to lawsuits that the court is competent to hear.

2. Standing Whether a specific person is the proper party to bring a matter to the court for adjudication. Requirements: 1. Injury Must allege they have suffered an injury or imminently will suffer injury. 2. Causation Must allege injury is fairly traceable to the defendants conduct. 3. Redress Must allege that a favorable decision is likely to remedy the injury. *Overlap between Causation and Redress. Allen v. Wright (1984) pg. 46 IRS is giving tax-exempt status to schools that are segregated. Injuries alleged: 1) the government is financing discriminatory practices. 2) Providing aid to segregated private schools impair their ability to have their public schools desegregated. 1) Fails because it does not constitute judicially cognizable injury. 2) Passes as an injury but fails because the injury is not fairly traceable to the assertedly unlawful conduct of the IRS (Causation). Lujaun v. Defenders of Wildlife (1992) pg. 62 Conduct of government agencies could result in the extinction of animals, injuring the plaintiffs ability to observe the animals. Court rejects the injury because it is not concrete enough. No plans to go to those places at that time. Only plans to go again in the future. The only constitutional issue is the Standing. The claim is statutory. The citizen suit provision of the ESA provides any person may commence a civil suit on his own behalf to enjoin any person including the US and any other governmental instrumentality or agency who is alleged to be in violation of any provision of this chapter. 16 USC 1540(g). *Standing, however is a constitutional requirement, which cannot be overridden by statute. City of Los Angeles v. Lyons (1983) pg. 60 Lyons seeking an injunction against the policy of LAPD to apply chokeholds. Lyons could show Injury, Causation, and Redress. For the injunction, he must show he will suffer the injury again in the future, which he cannot. Massachusetts v. EPA (2007) pg. 53 Mass. Wants the EPA to regulate carbon dioxide emissions of cars. Injuries: sea level could rise costing them 100s of millions of $ in damages. Causation (conceded by EPA) and redress is sufficient. Mass.s claim has Standing. States may receive more flexibility when suing Federal agencies. *** Congress cannot grant the ability for anyone to bring a lawsuit. However, A litigant to whom Congress accorded a procedural right to protect his concrete interestscan assert that right without meeting all the normal standards for redressability 3. Ripeness Whether a federal court can grant pre-enforcement review? Must demonstrate that harm has occurred or imminently will occur The fitness of the issue for judicial resolution. The case must be factually developed first. *Injury that could occur without review? 6

Poe v. Ullman (1961) pg. 92 Synopsis of Rule of Law: A penal statute is not ripe for constitutional challenge unless it is enforced by the state enacting the statute. Facts: The Connecticut Supreme Court of Errors construed a state penal statute as prohibiting the use of contraceptive devices and the giving of medical advice on their use. Appellants included a couple who had several pregnancies result with severely abnormal progeny which died shortly after birth, a couple whose wife had experienced a severely traumatic pregnancy and their physician, who believes the safest course of treatment for the couples includes using contraceptive devices. Issue: Is the Appellants claim ripe for judicial review? Held: No. Judgment affirmed. Connecticut has never attempted to fully prosecute any case under the statute. Because of this, not only have the Appellants not suffered injury in fact from the statute (injury), there is no evidence that they would be prosecuted for acting in violation of the statute (factually undeveloped). Dissent: (Douglas) argues that the mere threat of prosecution is injury in fact, that it is not the choice worthy of a civilized society to require individuals to risk penalty for their behavior to have their constitutional rights determined. Discussion: Although ripeness is the central issue in Poe, the Supreme Court does not articulate any clear guidelines to evaluate ripeness. Nonetheless, the Supreme Court seems to articulate that a penal statute that has not been enforced is not ripe for judicial review. Abbott Laboratories v. Gardner (1967) pg. 95 Synopsis of Rule of Law: Pre-enforcement review is appropriate where not prohibited by the text of the Act itself, nor inconsistent with the legislative intent behind the Act. There is an actual case or controversy where there has been a final agency decision and withholding judicial consideration will result in hardship to the parties. Facts: Congress amended the Federal Food, Drug and Cosmetic Act in 1962 to require manufacturers of prescription drugs to print the established name (generic name) of the drug prominently and in type at least half as large as the type used for the proprietary name (brand name) on labels and other printed material. The purpose was to inform doctors and patients of drugs established names so that they could be purchased at lower prices. The Commissioner of Food and Drugs published proposed regulations (in addition to the Act) which required all drug labels and drug advertisements to put the established name next to the proprietary name every time the proprietary name appeared. A group of 37 drug manufacturers (the Petitioners) challenged the regulations on the grounds that the Commissioner exceeded his authority under the Act in issuing the regulations. The District Court granted injunctive and declaratory relief against the Commissioner. The Court of Appeals for the Third Circuit reversed, holding (1) that pre- enforcement review of the regulations was not permitted by the Act, and (2) that no relief was available under the Administrative Procedure Act because no actual case or controversy existed. The Supreme Court of the United States granted certiorari. Issues: Did Congress, by its Federal Food, Drug and Cosmetic Act, intend to forbid pre-enforcement review of the sort of regulations promulgated by the Commissioner? Were the issues ripe for judicial decision? Would withholding court consideration result in hardship to the parties? Held: Reversed and remanded to the Court of Appeals to review the District Courts decision that the regulation was beyond the power of the commissioner. No. Nothing in the Act itself precludes pre-enforcement review. A review of the legislative history of the Act reveals that the specific review provisions were designed to provide an additional remedy, and not to cut down more traditional channels of review. The Act itself states, The remedies provided for in this subsection shall be in addition to and not in substitution for any other remedies provided by law. Yes, the issues presented 7

were ripe for judicial consideration, and withholding judicial consideration would result in hardship to the parties. The parties agreed that the issue tendered was a purely legal one. The regulations in issue were reviewable as a final agency action under the Administrative Procedure Act because when, as here, they are promulgated by order of the Commission and the expected conformity to them causes injury cognizable by a court of equity, they are appropriately the subject of attack. The regulations would have a direct day-to-day impact on the operation of the companies, who either had to incur huge costs to comply with the regulations requirements or risk prosecution. Dissent. There were two unpublished dissents by Mr. Justice Fortas and Mr. Justice Clark. Concurrence. None. Discussion: Courts should look to the text of the statute itself, along with the legislative history, to determine the intended application and scope. In this case, pre-enforcement review was not precluded by the Federal Food, Drug and Cosmetic Act. Pursuant to the Constitution of the United States, there must be an actual case or controversy in order for the Supreme Court of the United States to grant certiorari. Under the Administrative Procedure Act, final agency actions are considered ripe for judicial review. Here, the Petitioners would have suffered an operational and financial hardship if judicial consideration was withheld. 4. Mootness Any changes in fact or law while the suit is pending, bringing an end to the s injury. Exceptions: 1. Wrongs capable of repetition, but evading review. Roe v. Wade. Pg. 98 2. Voluntary Cessation voluntarily ceases allegedly improper behavior but is free to return it at any time. Pg. 100 3. Class action suits. Pg. 100 5. Political Question some Constitutional provisions are left to the political branches of government to interpret and enforce. Ex. Foreign Policy. Cases under the Guaranty Clause are non-justiciable. Gerrymandering is a practice that attempts to establish a political advantage for a particular party or group by manipulating geographic boundaries to create partisan, incumbent-protected districts. Malapportionment is characterized by an inappropriate or unfair proportional distribution of representatives to a legislative body. Marbury v. Madison Early proclamation of Political Question Doctrine. Derives from the separation of powers in the Constitution. Due to flexibility of the doctrine, it may seem to produce inconsistent results. Article IV 4 Guaranty Clause The United States shall guarantee to every State in this Union a Republican Form of Government, and shall protect each of them against Invasion; and on Application of the Legislature, or of the Executive (when the Legislature cannot be convened) against domestic Violence. Baker v. Carr (1962) pg. 104 In determining whether a question fall within the political question category, (1) The appropriateness under our system of government of attributing finality to the action of the political departments, and (2) Also the lack of satisfactory criteria for a judicial determination are dominant considerations. Claim Lack of population disparity in voting districts. Factors whether there is a political question: Political question is found where there is a: 8

(1) Textually demonstrable constitutional commitment of the issues to a coordinate political department; or (2) A lack of judicially discoverable and manageable standards for resolving; or (3) The impossibility of deciding without an initial policy determination of a kind clearly for nonjudicial discretion; or (4) The impossibility of deciding without an initial policy determination of a kind clearly for non judicial discretion; or (5) The impossibility of a court's undertaking independent resolution without expressing lack of the respect due coordinate branches of government; or (6) An unusual need for unquestioning adherence to a political decision already made; or (7) The potentiality of embarrassment from multifarious pronouncements by various departments on one question. *Court originally held non-justiciable but in a later case held that apportionment must meet the standard of one man, one vote; that all districts must be approximately equal in population size. In that case (Davis v. Bandemer), the Court held that political Gerrymandering cases are properly justiciable under the Equal Protection Clause. Have refused to hear any gerrymandering cases since. Commentary: If SCOTUS finds a political question and refuse to hear the case, they are actually finding (through constitutional analysis) that another branch acted within the bounds of their Constitutional powers. Political Question Doctrine Applied: i. Congressional Self-Governance Powell v. McCormack (1969) pg. 116 PQD Rejected ii. Presidents Conduct of Foreign Policy Goldwater v. Carter (1979) pg. 119 PQD Applied iii. Impeachment and Removal Nixon v. United States PQD Applied Nixon v. United States (1993) pg. 121 USDC Judge Impeached by Senate. Evidence hearing by a Senate sub-committee, then impeachment decided on by enter senate. Sole in the Constitution as to the senates impeachment power means the Senate alone can decide how to go about the impeachment process. Baker Factors 1) power was textually committed to the Senate by the Constitution. 2) What is the meaning of Try (Constitution Senate has sole power to Try impeachment).

THE FEDERAL LEGISLATIVE POWER ARTICLE I


CONSTITUTIONAL ANALYSIS: ACTS OF CONGRESS 1. Does Congress have the authority under the Constitution to legislate? 2. If so, does the law violate another Constitutional provision or doctrine, such as by infringing separation of powers or interfering with individual liberties. Article I Section 8 Clause 18 Necessary and Proper Clause To make all laws which shall be necessary and proper for carrying into execution the foregoing powers [of Section 8] Borrow money for the United States Regulate commerce Declare war Raise and support armies McCulloch v. Maryland (1819) pg. 129 Issue: (1) Does Congress have the power to charter a bank, and (2) can Maryland tax the Bank of the United States? Holding: (1) Yes, Congress has the power to charter bank. (2) Power to charter a bank is not an enumerated power in the Constitution. In addition to its enumerated powers, Congress has the power to make all laws that shall be Necessary and Proper. The power to tax is the power to destroy. The states have no power, by taxation or otherwise, to impede, burden, or control the operations of constitutional laws enacted by Congress. Reasoning: 1. Did the Congress have power to establish a national bank? a. This is very different than the Marbury opinion. Marshall speaks of the courts awful responsibility (whereas in Marbury it was easy, just an application of law). He speaks of the choice b/w hostility and peace, but this is not a legal consideration more like a political one. FEAR: that govt will not be powerful enough (impotency/paralysis). We can imply other powers that are not enumerated in the Constitution. We must never forget that it is a constitution we are expounding. Whereas in Marbury the nature of the Constitution was considered to be about providing strict limits, here he uses vague generalities to achieve its purpose (inferred from intent and language). b. New idea of Constitutional mission: govt should have means to achieve broad purposes. Turns upside down Marburys presumptions about the Consts mission (instead of limiting it, now want to liberate it). c. Necessary and proper clause: looks at how word is used by ordinary people. Words can be vague, contextual an anti-fundamentalist argument. An issue of degree, not of classification. Immutable rules would be unwise; desire for flexibility (to adapt to the various crises of human affairs). The standard is whether the govts means are adapted to its ends. Congress should have choice of means. Consequentialist argument: dont want to turn the Const into a [ornament]. 2. Did the state have the power to tax the bank? a. This half is much more like the Marbury opinion. The supremacy of the national govt is a great principle. This is the voice of certainty. FEAR: state tyranny. States are exercising power over non-constituents by taxing the national bank. b. Doesnt even look into the circumstances of the case to see whether the tax is disruptive; uses a bright line rule. We call for bright line rules when we should not defer to politicians because the matter is not one of degree. 10

Maryland Argument Constitution is an agreement between the States not the people, so states should have control over the federal government. N&P Clause is not granting powers, but is actually limiting powers, by the word necessary. Court responds that Necessary may some times mean Indispensible, but other times it is used figuratively, as convenient, or useful, or essential. 10th Amendment preserves the right of the sates to tax. Marshall responds, The right never existed, and the question whether it has been surrendered, cannot arise. McCulloch Argument Constitution is agreement amongst the people. States were a matter of practicality. CJ Marshall agrees. 10th Amendment The powers not delegated to the United States by the Constitution, nor prohibited by it to the states, are reserved to the states respectively, or to the people. Difference from clause in the Articles of Confederation is that it stated, Powers not Expressly delegated. Framers meant for Congress to have implied powers. Constitutional Interpretation: (C.J. Marshall) The Construction The Subject The Context The Intention of the person using them United States v. Comstock (2010) supp. pg. 15 Congress passes a federal statute that it can stay a federal prisoners release if they are a serious danger to society due to a mental illness. The argues that it violates the Due Process Clause (irrelevant to case). Question: Is this program permitted by the N&P Clause? Comstock Arg. When legislating pursuant to the N&P clause, Congress authority can be no more than one step removed from a specifically enumerated power. Court says, that argument is irreconcilable with their precedents. From the implied power to punish, it is further inferred both the power to imprison, and the federal civil-commitment power. *Supp. Pg. 21 How far does N&P extend? Court gives 5 Considerations as to where the causal link as to N&P Clause ends: 1. The breadth of the Necessary and Proper clause 2. The long history of federal involvement in this arena 3. The sound reasons for the statutes enactment in light of the governments custodial interest in safeguarding the public from dangers posed by those in federal custody 4. The statutes accommodation of the states interests 5. The statutes narrow scope (tailoring)

THE COMMERCE POWER


Article 1 8 Congress has the power to regulate commerce with foreign nations, and among the several states, and with the Indian tribes. 10th and 11th Amendment Limitations On Commerce Clause 10th: powers not delegated are reserved by the states. Congress can enact generally applicable laws. Garcia. Congress cant tell states what regulations to enact or take over their law-making mechanisms. New York v. U.S. Congress cant make state employees federal ones. Printz. 11

11th

Take-home lesson: Congress has right to do general stuff, but doesnt have the right to push responsibility for it off onto the states (take credit, mother fuckers). Commerce power ~allow Congress to abrogate a states sovereign immunity from suit. Seminole Tribe & Alden. a. Citizen suits may be okay under 5 of 14th Amendment, however, since it post-dates the 11th. i. When this is done, there must be congruence and proportionality. Kimel v. FL. b. But, Congress and government agencies can sue a state (e.g. for 14th Amendment violations) on their own. No real restriction here.

Founding -1890s Very broad federal powers to Commerce and N&P 1890s-1937 Contraction of powers, economy grows and becomes more integrated. 1937-1990s Court expansively defined the scope of commerce power 1990s-present narrowed the scope of commerce power and revived the 10th Amendment as an enforceable limit on federal actions.

Initial Era: What does Commerce mean? Gibbons v. Ogden (1824) pg. 142 Facts: NY legislature enacts statute granting Fulton and Livingston the exclusive right to operate steamboats in New York waters. Statute was designed to encourage investment in development of steamboat technology. Fulton/Livingston licensed Ogden to operate a ferry b/w NYC and NJ. Gibbons began operating a competing ferry service that, because it entered NY waters, violated the grant to Fulton and Livingston and the license to Ogden. However, Gibbons ferries were licensed as vessels in the coasting trade under a 1793 statute enacted by Congress (Federal License). Ogden tried to enjoin Gibbons. Issue: Did the Fed. Govt have the authority to issue the license, thus contradicting NY law. Holding: The 1793 statute authorized the entry of Gibbons ferries into NY waters. The NY monopoly was therefore invalid under the supremacy clause, and the injunction was dissolved. Reasoning: Congress has power to regulate commerce among the several states (any commerce that is not completely internal). (Regulating internal commerce would be inconvenient and unnecessary.) Congress can prescribe a rule by which commerce is to be governed. Marshall says that the commerce clause DOES include navigation commercial intercourse between nations. Ogdens argument that the states through the 10th Amendment have the power to regulate commerce with foreign nations and among the states because the power which the states gave up to Congress to regulate commerce was not absolute and the residue of that power remained with the states. In some cases, states can pass regulations which may affect some activity associated with inter-state commerce, but they must base such regulations on some other source of power rather than the commerce power Among means intermingled with. Power to regulate commerce does not extend to commerce that is completely internal within a state. Is this true in todays integrated economy?

12

1890-1937: Limited Federal Commerce Power What is Commerce? United States v. E.C. Knight Co. (1895) pg. 146 Congress trying to regulate a sugar refinery about to obtain control 98% of the sugar of the US industry. Court draws a distinction between manufacturing and commerce. Manufacturing comes long before commerce. No direct consequence to commerce. Manufacturing is not an area that can be regulated by Congress pursuant to the commerce clause. Direct vs. Indirect Test - Challenges to the Sherman Act launched new judicial limitations on Congressional authority over commerce. The court here took a narrow definition of commerce, and said what mattered was whether the regulated activity had a direct or indirect effect on interstate commerce here, the effect was indirect, so the court chose not to enforce the Sherman Act on this company. Carter v. Carter Coal Co. (1936) pg. 147 Effect of the labor provision falls upon production and the relation of employer and employee, both of with is a local relation. Such effect that they may have among commerce, however extensive it may be is secondary and indirect. This is an example of where the federal govt. might need to control something because states cant do it adequately/sufficiently, but that is not enough to bring the topic under federal power or the commerce clause. Rule: Just because a commodity is manufactured or produced within a state and is intended for interstate commerce, does not mean that its production or manufacturing is subject to federal regulation under the commerce clause. A commodity that is meant to be sold in interstate commerce is not considered to be part of interstate commerce before the commencement of its movement from the state. The evils that Congress sought to control are all local evils over which the federal government has no legislative control." The federal regulatory power ceases when interstate commerce ends; and, the power does not attach until interstate commercial intercourse begins. *The court can go either way & does what it wants* Dissent: (Cardozo) Claimed that the price-fixing provision of the Coal Conservation Act was within Congresss power because it had a direct effect on interstate trade. What does Among the States mean? Houston E. & W. T. Ry. Co. v. United States (1914) pg. 149 Congress authority, extending to these interstate carriers as instruments of interstate commerce, necessarily embraces the right to control their operations in all matters having A close and substantial relation to interstate traffic Rather than promoting states rights over federal power, SCOTUS perhaps means to prevent the federal government from overregulating the national economy. Substantial Economic Effects Test Congress was regulating intra-state railroad, the railroad was charging more for train hails between TX and LA, as compared to hauls which just went between points in Texas. The court held that since the effect was to discriminate against inter-state commerce, this comes under the Commerce Clause. Establishes a new test the substantial economic effects test which emphasizes the practical, physical or economic effects of the regulated intra-state activities on interstate commerce. A.L.A. Schechter Poultry Corp. v. United States (1935) pg. 151 Court rejects argument that just because >90% of s chickens came from out of state, it falls into 13

interstate commerce, because once the chickens were there they were prepared strictly for in state service. Also rejects that failure to apply minimum wage law has a substantial impact on interstate commerce, because lower costs means out of state suppliers would be adversely effected, because of distinction between direct and indirect effects, and thus Congress would be able to regulate all processes of production. When the stream of commerce begins and ends: The court invalidated regulations of the poultry industry according because it was (1) a violation of the non-delegation doctrine and (2) an invalid use of Congress's power under the commerce clause. The Court distinguished between direct effects on interstate commerce, which Congress could lawfully regulate, and indirect, which were purely matters of state law. Though the raising and sale of poultry was an interstate industry, the Court found that the "stream of interstate commerce" had stopped in this caseSchechter's slaughterhouses bought chickens almost exclusively from intrastate wholesalers and sold completely exclusively to intrastate buyers. Any interstate effect of Schechter was indirect, and therefore beyond federal reach. 10th Amendment separation of state and federal powers. 10th Amendment Limitations on Congressional Power. Hammer v. Dagenhart (1918) pg. 154 Facts: Congress enacted the Child Labor Act in 1916. It prohibited the transportation in interstate commerce of goods produced in factories employing children under age 14 or employing 14-to-16 year olds for more than eight hours a day, or six days a week, or at night. Two children, one under age 14 and the other under age 16, were employed in a cotton mill in NC. Their father got an injunction against enforcement of the Act on grounds of its unconstitutionality. Holding: The law goes beyond Congresss power to regulate interstate commerce b/c it seeks to regulate workplace conditions, which is the states authority. Invalidated. Reasoning: The Act in effect does not regulate transportation among the states, but aims to standardize the ages at which children may be employed in mining/manufacturing within the states. Commerce begins only when a product is actually delivered to a common carrier for transportation or [at] the actual commencement of its transfer to another state. The Commerce Clause was not intended to give Congress a general authority to equalize working conditions. The goods are not harmful in themselves (classification!). To sustain this statute would be to sanction an invasion by the federal power of the control of a matter purely local in its character, and over which no authority has been delegated to Congress in conferring the power to regulate interstate commerce. Regulating children in the workplace is a state authority. FEAR: slippery slope: if Congress can regulate local matters in this way, all freedom of commerce will be at an end, and the power of states over local matters may be eliminated, and our system of govt practically destroyed. Dissent: (Holmes) If an act is within the powers specifically conferred upon Congress, it seems that it is not made any less constitutional because of the indirect effects that it may have. The Act does not meddle with anything belonging to the states. They can regulate their internal affairs as they like. But when they seek to send their goods across the state line, they are no longer within their rights. Distinguished from Champion by The goods shipped are of themselves (morally) harmless middle of pg. 155. Champion v. Ames (1903) pg. 158 The intention of the law is of no relevance, (morality of gambling). Members of Congress may have different intentions. 14

The congressional act is controlling the movement of items between the states. Thus it falls under Congress power to regulate commerce. The court held that trafficking lottery tickets constituted interstate commerce that could be regulated by the U.S. Congress under the Commerce Clause. Most important in this case was that the Supreme Court recognized that Congress' power to regulate interstate traffic is plenary. That is, the power is complete in and of itself. Rule: This wide discretion allowed Congress to regulate traffic as it sees fit, within Constitutional limits, even to the extent of prohibiting goods, as here. That means that once you are clearly IN interstate commerce, Congress can regulate for any reason, even non-economic moral reasons. Commentary: Roosevelt, as part of the new deal wants to increase # of SCOTUS justices, in order to pass New Deal legislation. This is a power of Congress. Congress rejects a bill that would allow the appointment of justices with each new president, thus stacking the court politically, and changing the interpretation of the constitution with each new president. SCOTUS starts to read congressional power more broadly, approving New Deal legislation. 1937-1990s: Broad Federal Commerce Power NLRB v. Jones & Laughlin Steel Corp. (1937) pg. 160 Facts: The National Labor Relations Act established a comprehensive system for regulating labor/mgmt relations. It established the right of employers to organize and bargain collectively and created a board to supervise elections and enforce the acts prohibition of unfair labor practices like discrimination against union members. The act made a bunch of findings that these issues burden and obstruct commerce. J&L is a large steel producer with parts of its operations in different states. The NLRB (the Board created by the act) charged J&L w/the unfair labor practice of firing employees for trying to start a union. J&L challenged the constitutionality of the Act. Holding: The Act is valid. Although activities may be intrastate in character when separately considered, if they have such a close and substantial relation to interstate commerce that their control is essential or appropriate to protect that commerce from burdens and obstructions, Congress may regulate them. Reasoning: Acts which directly burden or obstruct interstate or foreign commerce, or its free flow, are within the reach of the congressional power. It is the effect on commerce, not the source of the injury (e.g., labor disputes), which is the criterion. Employees have a clear right to organize; discrimination is thus a proper subject of regulation by competent legislative authority. J&L argues that manufacturing in itself is not commerce, but the various parts of J&Ls enterprise are interdependent. The congressional authority to protect interstate commerce from burdens/obstructions is not limited to transactions which are deemed an essential part of a flow of interstate commerce. The questions involved here are all questions of degree. It doesnt matter what the employees here were doing (what was the activity regulated), we only look at its effect on commerce. The effect would be immediate and catastrophic if the activity were to stop. We cannot ignore actual experience- judgment grounded in reality. Rule: Under the Commerce clause, Congress has the power to regulate any activity, even intra-state production, if the activity has an appreciable effect, direct or indirect, on inter-state commerce. There must be a close and substantial relationship to interstate commerce such that their control is necessary to protect commerce. United States v. Darby (1941) pg. 164 Facts: Darby was charged with violating the Fair Labor Standards Act of 1938. The act prohibited the shipment in interstate commerce of goods manufactured by employees who were paid less than a prescribed minimum wage or who worked more than a prescribed maximum number of hours and 15

prohibited the employment of workers in production for interstate commerce at other than the prescribed wages and hours. Darby objected to the constitutionality of the act. Holding: The Act is valid. The Const does not in any way restrict the motive/purpose of a regulation of interstate commerce. (Overrules Hammer) Reasoning: While manufacture is not of itself interstate commerce, the shipment of manufactured goods interstate is such commerce and the prohibition of such shipment by Congress is indubitably a regulation of the commerce. Congress is free to exclude from commerce articles whose use in the destination states may cause injury to public health/morals/welfare. The regulation of interstate commerce is itself a legitimate end. Regulation of intrastate activities is fine if those activities so affect interstate commerce as to make regulation of them appropriate means to the end of regulating interstate commerce. Discussion: The only question is whether the intrastate activity has an effect on other states a question of fact/degree. Deference to legislature. Sweeps away the idea that the national govt cant redistribute economic advantage among the states. In theory, this Act would be okay even if it didnt prohibit interstate treatment. The bootstrap argument: if it can regulate interstate commerce on these grounds, it can also directly regulate the intrastate activities to boost its program. If activities intrastate have a substantial effect on interstate commerce, then Congress has the power to regulate it. Darby says the 10th Amendment is a truism, not an express reservation of anything extra. All it says is that the states get the federal governments leftovers. So the only question to ask is whether the regulation is within federal power. If yes, then you dont really focus on consequences to state power. 1941 *10th Amendment only states a truth that the government is one of enumerated powers, and if a power is granted by the Constitution then there is no other restriction granted by the 10th Amendment.* Wickard v. Filburn (1942) pg. 166 Facts: Under the Agricultural Adjustment Act, the Secretary of Agriculture set a quota for wheat production after finding that the total supply of wheat would exceed a normal years consumption and export needs. The quota was approved in a referendum of wheat growers. Each wheat grower was given an allotment. Filburn had a dairy farm in Ohio on which he raised small amounts of wheat for his livestock, for making flour at home, and for sale. He harvested too many bushels and was fined. Holding: The Act (and penalty) are valid under the aggregation principle. Reasoning: This Act extends federal regulation to production not intended in any part for commerce but wholly for consumption on the farm. We dont want to categorize things as direct/indirect anymore; we just want to look at the actual effects of the activity upon interstate commerce. No mechanical application of formulas. Even if Filburns activity was local and not regarded as commerce, it may still be reached by Congress if it exerts a substantial economic effect on interstate commerce. That his own contribution to the demand for wheat may be trivial by itself is not enough to remove him from the scope of federal regulation where, as here, his contribution, taken together with that of many others similarly situated, is far from trivial. Broad govt power is ok quotes Gibbons. Discussion: This is the aggregation argument. After this, there was almost no protection from regulation by the national govt. fined for growing wheat great then the amount established under federal law. The wheat was grown not for commercial purposes, but for his family to use. Court says it doesnt matter. May still 16

be reached by Congress if it exerts a substantial economic effect on interstate commerce, regardless of direct or indirect effect. Aggregation Principle one man doing this is not a substantial effect, but when you add all the people that are similarly situated that may do the same, it would have a substantial economic effect. Heart of Atlanta Motel, Inc. v. United States (1964) pg. 169 Facts: Title II of the 1964 Civil Rights Act provides that all persons shall be entitled to the full and equal enjoyment of the goods, services, and accommodations of any place of public accommodation w/o discrimination or segregation on the ground of race, color, religion, or national origin. It defines places of public accommodation as those whose operations affect commerce. It declares that hotels and motels that provide rooms for transient guests affect commerce per se. A restaurant is covered if it serves or offers to serve interstate travelers or if a substantial portion of the food which it serves has moved in commerce. Heart of Atlanta Motel wanted the law overturned. The motel advertised in national magazines and on billboards; about 75% of its guests were from out of state. Holding: The Act is valid. The power of Congress to promote interstate commerce also includes the power to regulate the local incidents thereof, including local activities having a substantial and harmful effect on that commerce. Reasoning: There is a qualitative (deterrence of Black interstate travelers) as well as quantitative effect on interstate travel by black people. Racial discrimination thus has a disruptive effect on commerce. It does not matter that the motel was of a purely local character b/c Congress can regulate local incidents of interstate commerce. How obstructions in commerce may be removed (what means to be employed) is within the sound and exclusive discretion of Congress. It is subject only to one caveat that the means chosen by it must be reasonably adapted to the end permitted by the Const. We cannot say that its choice here was not so adapted. Discussion: Practically no limit now on what Congress can regulate. Congress enacts this law under their Commerce powers, not Equal Protection. The fact that this is a moral law is once again irrelevant (Champion) because it falls within Congress Commerce power. Katzenbach v. McClung, Sr. & McClung, Jr. (1964) pg. 171 Facts: Ollies BBQ was a family-owned restaurant in Alabama located on a state highway 11 blocks from an interstate. The restaurant caters to a family and white-collar trade w/a take-out service for black people. It employs 36 persons, two thirds of whom are black. The restaurant bought about $150,000 worth of food, of which $70,000 worth was meat bought from a local supplier who purchased it from out of state. The restaurant challenged the constitutionality of Title II of the CRA (see Heart of Atlanta). Holding: The Act is valid. If a law has a Rational Basis towards protecting interstate commerce, the only question left to ask is whether the restaurant serves or offers to serve interstate travelers; this one does. Reasoning: Discrimination in restaurants has a direct and highly restrictive effect on interstate travel by black people. It causes restaurants to sell less interstate goods, thereby obstructing the commerce. Viewed in isolation, the amount of food is insignificant, but you can aggregate. Concurrence: (Black) Not every remote, speculative effect on commerce should be accepted as an adequate constitutional ground to uproot and discard all our traditional distinctions b/w what is purely local (and controlled by state laws) and what affects the national interest (controlled by federal laws). The aggregation principle helps us figure out what to do in a given case. Local restaurant purchased food from a local supplier that purchased from out of state suppliers. Court Holding: It is enough that congress concluded that there was a substantial effect. Only for the court to decide that Congress had a Rational Basis for finding a substantial effect on interstate 17

commerce. - Lowers the threshold grants discretion to the legislature, the court should defer to the expertise of congress. To what extent does the 10th amendment restrict Congress power? 1976 Revival of idea that 10th amendment is a separate restriction on Congressional power. National League of Cities v. Usery (1976) pg. 175 Overruled by Garcia The Court held that the Commerce Clause did not empower Congress to enforce the minimum wage and overtime provisions of the Fair Labor Standards Act against the states in areas of traditional govt functions. The Court agreed that the wages and hours of state employees affected interstate commerce but found the application of the statute to state and local employees unconstitutional. Discussion: Rehnquist said states are sovereigns. They make policy, but to do that, they have to be in control of their own processes/internal functions. They needed proprietary self-control, and the 10th Am was held to protect that. NO substantial intrusion on internal self-mgmt. This lacks the appeal of protecting specific policy areas. Fair Labor Standards Act deals directly with state employees. Supposed interference with the states sovereignty power to structure employer-employee relationships. Draws Distinction between traditional and nontraditional state government functions. Congress cannot interfere with traditional functions but can with nontraditional. Garcia v. San Antonio Metropolitan Transit Authority (1985) pg. 178 Holding: Usery is overruled. No traditional govt functions test. Reasoning: Govt function is whatever the people want it to be. So, that test disserves principles of democratic self-governance. Dissent: (Powell) Most effective democracy occurs at local levels of govt (consequentialist argument). (OConnor) States do have traditional interests. Discussion: Calls for return to post Lochner era no Const. law limiting the scope of federal intrusion on state/local prerogatives. Political safeguard argument: no NEED for Const. protection b/c states can protect themselves. This argument is questionable. The question of whether states DESERVE Const. protection is more interesting. After this setback, a shift in strategy. Attempt to draw the boundaries of state regulatory immunity in terms of traditional and nontraditional governmental functions is unworkable and also inconsistent with established principles of federalism. The court has held ordinarily the state interests are more properly protected by the inherent restraints of state participation that are built into the structure of the federal system (e.g. state representation in Congress) than by judicial review. Rule: Thus, at least absent a showing of some extraordinary defects in the procedural safeguards for the states in the national political process, the court will not find that an otherwise permissible exercise of Congressional power violates the 10th Amendment merely because it regulates the states as well as private persons. 1990s-Present Narrowing of the Commerce Power & Revival of 10th Amendment Restraint on Congressional Authority United States v. Lopez (1995) pg. 184 Facts: In the Gun-Free School Zones Act of 1990, Congress made it a federal offense for any individual knowingly to possess a firearm at a place that the individual knows, or has reasonable cause to believe, is a school zone. Holding: The Act exceeds the authority of Congress to regulate interstate commerce. 18

Reasoning: The Commerce Clause power is subject to outer limits, notwithstanding Jones & Laughlin, Darby, and Wickard. Under its commerce power, Congress can regulate (1) the use of the channels of interstate commerce, (2) the instrumentalities of interstate commerce or persons/things in interstate commerce, even though the threat may come from only intrastate activities, (3) activities having a substantial relation to interstate commerce i.e., those activities that substantially affect interstate commerce. This Act has nothing to do w/ commerce or any sort of economic enterprise. The govt argues that possession of a firearm in a school zone results in violent crime and this crime can be expected to affect the functioning of the national economy. This is a slippery slope: under that reasoning, Congress could regulate all activities tenuously relating to crime, or even any activity that it found was related to economic productivity of citizens: family law for example. We need limits. We cant let ANY activity be looked at as commercial. It has to be an economic activity. Concurrence: (Kennedy) This content-based boundary is pretty imprecise. Also, hes concerned about the Court itself although affirms that the judiciary can play a role here even though commerce clause concerns are inevitably political. He also asks whether the fed govt is intruding on an area of traditional state concern (the old approach majority doesnt do this). (Thomas) Disagrees w/the substantially affect standard. It has no stopping point b/c of the aggregation principle. Its going to allow the fed govt to regulate too many things. We need a bright line. Dissent: (Stevens) Guns are articles of commerce, and articles that can be used to restrain commerce. Their possession is the consequence of commercial activity. (Souter) Lets just use rational basis review. There was a rational basis for finding that this subject substantially affects interstate commerce. Grading things as directly/indirectly affecting commerce looks too much like Lochner. (Breyer) The commerce connection is not a technical legal conception but a practical one. Congress could reasonably have found an empirical connection here. No slippery slope fears needed, this is a rare case. Discussion: The court moves from the rational relationship test back to the Lochner era substantially affect test. Slippery slope arguments were a Lochner era staple as well. Builds in a classification commerce must be an economic activity or trade. What are some other possible arguments against the gun law? McCulloch concept that if Congress exercises power in a way that looks within its commerce clause power but is actually just a pretext, might strike it down. (This requires looking into motivation, and would require re-rationalizing the civil rights cases.) Gibbons idea that at one point an article is in commerce and at another point its at rest, and here the guns are at rest. 1st time SCOTUS has struck down a federal statute as beyond the scope of Congresss Commerce power since before 1937. Guns in a school zone is Not an economic activity. Court says that it will NOT allow aggregation of Noneconomic activities in substantial effect test. Does not say Congress cannot regulate noneconomic activities that substantially effect commerce. 3 areas of Congressional power under commerce clause: 1. Use of channels of interstate commerce. 2. Regulate and protect the instrumentalities of interstate commerce, or persons or things. 3. Activities that substantially affect/relate interstate commerce. Factors in determining whether legislation represents a valid effort to use the Commerce Clause power to regulate activities that substantially affect interstate commerce: 1. Whether the activity was non-economic as opposed to economic activity; previous cases involved economic activity. 2. Jurisdictional element (Was the law limited to a discrete set of firearms that additionally have an explicit connection with or effect on interstate commerce?) Are the means tailored to the ends to limit intrusion into state power? 19

3. Whether there had been Congressional findings of an economic link between guns and education. 4. How attenuated the link was between the regulated activity and interstate commerce. Issue: What is the limit on Congresss commerce power? Majority Substantial affect test does not apply to the aggregation of noneconomic activities. Concurrence Substantial Affect test/aggregation principle gives Congress unlimited police power, Court should return to Pre-New Deal jurisprudence Dissent Rational Basis Test Dissent says that everything has an economic aspect. Must ask what is the impact on interstate commerce If so, where is the limit on Congresss commerce power. Commentary: Post-opinion, Congress rewrites the questioned statute to fall under the 2nd area of commerce power rather than the 3rd. Reaffirmed in Dissent of Alderman v. U.S. (2011) 1990s-Present Narrowing of the Commerce Power United States v. Morrison (2000) pg. 196 Facts: The Violence Against Women Act provided a damage remedy for the victim against any person who commits a crime of violence motivated by gender. The statute contained detailed findings that gender-motivated violence affects interstate commerce by deterring potential victims from traveling interstate, engaging in employment in interstate business, etc. Holding: The damages provision is unconstitutional. Gender-motivated crimes of violence are not economic activity. Reasoning: Slippery slope: allowing this would let Congress regulate ANY crime as long as the nationwide, aggregated impact of that crime has substantial effects on employment, production, etc. The Const. requires a distinction b/w what is truly national and truly local (bright line classification!). Dissent: (Souter) This is categorical formalism. Circumstances have changed. (Breyer) Why should the economic or noneconomic nature of an interstate-commerce-affecting cause have constitutional importance? These random distinctions do little to further the federalism the majority wants to promote. Federal statute creating a civil cause of action for violent crimes motivated by gender. Based under 3rd area of commerce power activities that substantially affect interstate commerce. U.S. Argues substantially affected by violent crimes motivated by gender in that the aggregate reduces the amount out of state travelers willing to travel. Gender based crimes are Noneconomic activities, so the aggregation principle cannot apply. Difference from Lopez is that this act was supported by numerous congressional findings. Analysis Court says they are going through the Rational Basis test. 1. Activity was Noneconomic, cannot be aggregated. 2. Jurisdictional element: NO. Federal cause of action was not in the pursuance of Congresss power to regulate interstate commerce. 3. Whether there had been Congressional findings of an economic link between gender based crime and commerce? Yes, but not sufficient by itself. 4. How attenuated the link was between the regulated activity and interstate commerce? Congresss findings rely on a method of reasoning that we have already rejected Aggregate substantial affect. *Congressional regulation of intrastate activity effecting interstate commerce has always been economic in nature.

20

Gonzales v. Raich (2005) pg. 205 Facts: Federal law banned the private cultivation and use of marijuana, as applied to marijuana grown at a persons home and intended solely for use by that person or a person for whom the grower was a caretaker, as a medical treatment, legal under state law. Holding: The ban is constitutional. Reasoning: Congress can regulate purely intrastate activity that is not itself commercial, in that it is not produced for sale, if it concludes that failure to regulate that class of activity would undercut the regulation of the interstate market in that commodity. Congress had a rational basis for concluding that the high demand in the interstate market would draw the marijuana into that market. Activity here is an economic activity. Concurrence: (Scalia) The regulation of an intrastate activity may be essential to a comprehensive regulation of interstate commerce even though the intrastate activity does not itself substantially affect interstate commerce. That Possession is a noneconomic activity is immaterial to whether it can be prohibited as a necessary part of a larger regulation. Commerce clause plus Necessary & Proper clause. Dissent: (OConnor) Disagreed w/factual premise. No evidence to show that homegrown medicinal marijuana users constitute, in the aggregate, a large enough group to have a substantial impact on the national illicit drug market or to threaten the overall regulatory scheme. Discussion: This relates to the Darby bootstrap doctrine. Federal Drug Abuse Prevention Act makes it unlawful to manufacture, distribute, dispense, or possess marijuana. Difference between Gonzales (as applied) & Lopez/Morrison (facial) Facial Challenge challenge to the constitutionality as a whole. More difficult As Applied Challenge challenge to the application of the statute. Unconstitutional to a specific group of persons, but not as a whole. Seeks to make an exception. Act applied to persons under the act doing so for medicinal uses, is unconstitutional. Analysis: 1. Activity was Economic, irrelevant that it was illegal because there is still a huge market. Possession of marijuana economic activity? Lopez possession of guns noneconomic? 2. Jurisdictional element: 3. Whether there had been Congressional findings of an economic link between gender based crime and commerce? Yes, but not sufficient by itself. 4. How attenuated the link was between the regulated activity and interstate commerce? Congresss findings rely on a method of reasoning that we have already rejected Aggregate substantial affect. Florida v. U.S. Dept. of Health and Human Services, N.D. Fla. (2011) It would be a departure from case law to hold that Congress can regulate inactivity under the Commerce Clause. If it has that Congress could do almost anything it wanted Under this logic, Congress could require that people buy and consume broccoli. The individual mandate falls outside the boundary of Congress Commerce Clause authority By definition, it cannot be proper. (i.e. Necessary and Proper) Congress cannot regulate Inactivity that has an effect on interstate commerce. *Yet to be decided by SCOTUS.

21

Revival of 10th Amendment Restraint on Congressional Authority New York v. United States (1992) pg. 216 Facts: The 1980 Act declared that each state was primarily responsible for disposing of low-level radioactive waste produced within its borders and authorized states to enter into regional compacts for disposal facilities. The 1985 Act gave three incentives to induce states to provide for such disposal: (1) Monetary (refund of the surcharge), (2) Access (escalating surcharges for access to existing disposal sites), and (3) Take Title Provision (if a state was unable to provide for disposal of its own waste by 1996, it was to take title to the waste and would be obligated to take possession of the waste. Holding: While Congress has substantial power under the Const. to encourage states to provide for the disposal of the radioactive waste generated within their borders, the Const does not confer upon Congress the ability simply to compel the states to do so. Reasoning: Congress cant commandeer the legislative processes of the states by directly compelling them to enact and enforce a federal regulatory program. The Const gave Congress the power to regulate individuals, not states. Even where Congress has the authority to pass laws requiring/prohibiting certain acts, it cannot directly compel the states to require/prohibit those acts. Congress can encourage through incentives, it can condition the spending of federal funds, it can engage in cooperative federalism. In all those methods, the residents of the state retain the ultimate decision as to whether or not the state will comply. When the fed govt forces the state govt to do something, accountability is diminished b/c state officials will get blamed when fed officials devised the program. The first and second set of incentives under this scheme are OK. They allow state choice. But the third set is not OK. It offers two choices, either one of which if forced on the state separately would be impermissible. No matter which choice the state chooses, it has to follow the direction of Congress (take title or agree to regulate). A strong federal interest doesnt change the outcome. State consent also doesnt change the outcome the purpose of the separation of fed/state power is to protect individuals. Dissent: (White) This program is about cooperative federalism. A failing of the political process cant be shown here; the majority refuses to rebut the argument that the states were able to look after themselves leading up to passage of the Act. FEAR of tyranny is misplaced; what we need to FEAR now is impotent govt unable to deal with the problem of radioactive waste. (Stevens) The federal govt can command the states as it pleases. Discussion: Parker argues that the accountability argument seems superficial, but legal conservatives have seized on it. Also a weak spot in this case is compel: fed govt can get around this by using incentives, preempting, etc. It just cannot commandeer state processes. Constitutional line between federal and state power. 2 ways to frame issue: Whether an act of Congress is authorized by one of the powers delegated to Congress by the Constitution. Whether an act of Congress invades the province of state sovereignty reserved by the 10th amendment. Printz v. United States (1997) pg. 226 Facts: The Brady Act required the Attorney General to establish a national instant background check system by 1998. Until then, gun dealers were required to send a form identifying a purchaser to the chief law enforcement officer (CLEO) of a prospective purchasers residence. The dealer then had to wait 5 days to complete the sale. When the CLEO received the form, the CLEO had to make a reasonable effort to ascertain whether receipt or possession would be in violation of the law. Holding: No good under New York. The statute compels enlistment of state officers for the administration of federal programs. 22

Reasoning: The Const doesnt speak on this question, so were looking at historical understanding and practice, the structure of the Const, and precedent. New York applies not just to state legislatures but also to state executive officials, because the reasons for protecting both of them from commandeering are the same. We need a precise barrier against fed intrusion on state authority. Dissent: (Stevens) Where Congress exercises the powers delegated to it by the Const, it may impose affirmative obligations on exec and judicial officers of state and local govt. Historical materials show that the founders intended to enhance the power of the fed govt by empowering it to act through local officials. The accountability argument sucks b/c if somethings politically unpopular, you can be sure that politicians will let you know exactly who was responsible for it. Federal ways to influence states: (1) Federal Funding (2) Cooperative Federalism

CONGRESS TAXING AND SPENDING POWER


Article I 8 Congress shall have the Power To lay and collect Taxes, Duties, Imposts and Excises, to pay the Debts and to provide for the common Defense, and general welfare of the United States; but all Duties, Imposts and Excises shall be uniform throughout the United States. United States v. Butler (1936) pg. 239 Old Rule Facts: The Agricultural Adjustment Act of 1933 stated that there was a national economic emergency arising from the law price of agricultural products in comparison with other commodities. To remedy the situation, a tax would be collected from processors of an agricultural product. The revenue raised would be paid to farmers who reduced their production of that product. Rule: Congress does have broad authority to tax and spend for the general welfare, but may not, under the pretext of exercising the taxing power accomplish prohibited ends, such as the regulation of matters of purely state concern and clearly beyond its national powers (10th Am. limitation). Madison v. Hamilton: Madison said that the grant of power to tax and spend for the general national welfare must be confined to the enumerated legislative fields committed to the Congress (mere redundancy). Hamilton maintained the clause maintains a power separate and distinct from those later enumerated, is not restricted in meaning by the grant of them, and Congress consequently has a substantive power to tax and to appropriate, limited only by the requirement that it should be exercised to provide for the general welfare of the U.S. South Dakota v. Dole (1987) pg. 245 Overrules Butler limitation Congress passed legislation, the National Minimum Drinking Age Act, designed to discourage states from lowering the legal drinking age in 1984. It did so by withholding 5% of federal highway funding from states that did not comply. In 1988, that amount changed to 10%. South Dakota, which had allowed 19-year-olds to purchase beer containing up to 3.2% alcohol, sued to challenge the law, naming Secretary of Transportation Elizabeth Dole as the defendant because her office was responsible for enforcing the legislation. Court decided that Congress applying its taxing and spending clauses was not in violation of the 21st Amendment. It is subject to four covenants: 1. Exercise of spending power must be in pursuit of the general welfare. 2. If Congress is going to condition the state funding, it must make clear to the states what the consequences of participation are (unambiguous conditions so states can sign on to it knowingly). 3. Conditions must be related to the federal interest in the particular national project: Germane requirement: the condition cant be something totally unrelated to the federal interest in the particular program, must be germane, there has to be this nexus

23

4. Other constitutional provisions may act as independent bar to conditional grant of federal funds. Limitation: Congress cant condition its funding on state doing something affirmatively unconstitutional, meaning not just reserved to the states (i.e. cant condition paying the states on state banning abortion); reserved powers are on a different plain than other affirmative powers The first 3 restrictions were uncontested. The 10th Amendment bars federal regulation of the States, and it has been suggested that the Twenty-First Amendment might prohibit federal regulation of the drinking age. Nevertheless, the Congressional condition of highway funds is merely a "pressure" on the State to comply, not a "compulsion" to do so, because the State's failure to meet the condition deprives it of only 5% of the highway funds it may obtain. Highway funding is related to the condition in that if states have different drinking ages, those who are affected by it will use the highways to go to a state with a lower age, and drive on the highways drunk. Therefore, Congress did not run afoul of the 10th or 21st Amendments. Rule: Congress may place strings on state funding, so long as the conditions are expressly stated and so long as they have a relationship to the purpose of the spending program. Justification for Limitations on conditional funding: Congress can use its spending power to manipulate state laws and go beyond its enumerated power.

13th Amendment 1 Neither slavery nor involuntary servitude, except as a punishment for crme whereof the party shall have been duly convicted, shall exist within the United States, or any place subject to their jurisdiction. 2 Congress shall have power to enforce this article by appropriate legislation. (Include Necessary & Proper powers Jones v. Alfred H. Mayer Co.,) 14th Amendment 1 All persons born or naturalized in the United States, and subject to the jurisdiction thereof, are citizens of the United States and of the State wherein they reside. No State shall make or enforce any law, which shall abridge the privileges or immunities of citizens of the United States; nor shall any State deprive any person of life, liberty, or property, without due process of law; nor deny to any person within its jurisdiction the equal protection of the laws. 5 Congress shall have power to enforce this article by appropriate legislation. (Include Necessary & Proper powers Katzenbach v. Morgan & Morgan) 15th Amendment 1 The right of citizens of the United States to vote shall not be denied or abridged by the United States or by any State on account of race, color, or previous condition of servitude. 2 Congress shall have power to enforce this article by appropriate legislation. (Include Necessary & Proper powers Katzenbach v. Morgan & Morgan) Whom can be regulated under Post-Civil War Amendments? Civil Rights Cases (1883) pg. 248 Facts: Civil Rights Act of 1875 provided that all persons were entitled to full and equal enjoyment of the accommodations, advantages, facilities, and privileges of inns/theatres/places of public amusement, subject only to conditions and limitations established by law, and applicable alike to all citizens of every race and color, regardless of previous condition of servitude. Private persons violating these rights were subject to civil damages and criminal penalties. The Acts constitutionality was challenged on the ground that it was not authorized by any substantive grant of power to the federal govt. Holding: The 14th Amendment does not authorize the govt to restrict private action. Not authorized by the 13th Amendment. Discrimination described by the act has nothing to do with slavery or indentured servitude. Reasoning: Congress did not refer to state action when passing the act. The wrongful acts of individuals cannot impair constitutional rights. Acts of refusal have nothing to do with slavery; just 24

CONGRESSIONAL POWER UNDER THE CIVIL RIGHTS AMENDMENTS

an ordinary civil injury. At some point, people have to become mere citizens & stop being special favorites of the law. Dissent: (Harlan) Fundamentalist approach: the assumption that the 14th Am consists wholly of prohibitions upon state laws/proceedings is unauthorized by its language. The intent of the amendments was to protect people against deprivation, because of their race, of any civil rights guaranteed to free men. Discussion: Fear: national govt tyranny over state/local govt and political processes. When Bradley says that injured individuals can go to the state for redress, this is an absurdity in 1883. -Counterargument to assertion that 14th Am only restricts state action: the state can impair rights through inaction (by failing to stop private discrimination) -The text is interpreted restrictively (fundamentalist). Civil Rights Act of 1875 prohibited private racial discrimination by hotels, restaurants, transportation, and other public accommodations. Act not authorized by 13th or 14th Amendments. Until some state law has been passed or some state action through its officer or agent has been taken no legislation of the U.S. under the 14th amendment can be called into activity, for the prohibitions of the amendment are against state laws and acts done under state authority. *Authority granted by 14th Amendment does not reach private action, only state action. Jones v. Alfred H. Mayer Co. (1968) pg. 249 1982 which bars all racial discrimination, private as well as public, in the sale or rental of propertyis a valid exercise of power of Congress to enforce the 13th Amendment. Congress is authorized by the 13th Amendment to pass all laws necessary and proper for abolishing all badges and incidents of slavery in the United States. 13th Amendment 2 (Enabling Clause) authorized Congress to prohibit private acts of discrimination as among "the badges and incidents of slavery." Congress possessed the power to "determine what are the badges and incidents of slavery, and the authority to translate that determination into effective legislation necessary & proper (appropriate) for abolishing them." United States v. Morrison (2000) pg. 250 Reaffirms civil rights cases with regard to the 14th amendment and Congresss lack of authority to regulate private behavior. Congress may address private discriminatory conduct, subject to certain limitations. Holding: Congress may address discriminatory conduct through 5 of the 14th amendment (subject to certain limitations, but if congressional laws are directed toward conduct of private persons, the laws exceed Congress limits. Dissent: The statute being challenged regulates private action indirectly of regulating the States conduct in not providing sufficient remedies to victims. Remedies of the private action are proportional and congruent to the regulation of state conduct. Private action is Remedial and preventive as it relates to the state. *Congress can reach and regulate private action indirectly, so long as it is proportional and congruent to the regulation of states conduct. Commentary on 13th & 14th Power: Congress can prohibit private racial discrimination under 2 of 13th Amendment. Congress can only attack discriminatory conduct by the States under 5 of 14th Amendment and NOT private conduct. *********

25

Scope of Congressional Power under Post-Civil War Amendments 2 views as to scope of Congresss power under 13th, 14th, & 15th Amendments enforcement clauses 1. Narrow Accords Congress only authority to prevent or provide remedies for violations of rights recognized by the Supreme Court. 2. Broad Accords Congress authority to interpret the 14th Amendment to expand the scope of rights or even to create new rights. (Even if activity is not itself unconstitutional, Congress may still be able to regulate it). Katzenbach v. Morgan & Morgan (1966) pg. 253 *Broad Scope Upholds the constitutionality of a provision of the Voting Rights Act of 1965. Argument for Narrow view made that act violates the 14th amendment because precedent provides that literacy test as a precondition to voting is constitutional. For the provision to be constitutional, the application of the literacy test prohibited by the provision must be outlawed by the Supreme Court; which precedent shows? Court reads act under Broad approach. 5 of 14th amendment grants Congress the same broad powers expressed in the Necessary and Proper Clause. It is enough that the Court be able to perceive a [Rational] Basis for Congresss determination. 5 Considerations as to where the causal link as to N&P Clause ends: 1. The breadth of the Necessary and Proper clause 2. The long history of federal involvement in this arena 3. The sound reasons for the statutes enactment in light of the governments custodial interest in safeguarding the public from dangers posed by those in federal custody 4. The statutes accommodation of the states interests (tailoring) 5. The statutes narrow scope Employment Div., Dept. of Human Resources of Oregon v. Smith. (1990) pg. 257 Distinguished from Sherbert. Held that the Free Exercise Clause cannot be used to challenge neutral laws of general applicability (not motivated by a desire to interfere with religion). City of Boerne v. Flores (1997) pg. 258 *Narrow Scope Facts: The RFRA prohibits govt from substantially burdening a persons exercise of religion even if the burden results from a rule of general applicability, unless the govt can demonstrate the burden (1) is in furtherance of a compelling govt interest, and (2) is the least restrictive means of furthering that compelling govt interest. The Acts mandate applies to any branch, dept., agency, instrumentality, or official of the U.S. as well as to any state or subdivision of a state. Holding: The statute exceeds Congresss power. Reasoning: Congress has power to enforce 14th Am provisions under 5 of that Am. Therefore it has the power to enforce the free exercise clause of the 1st Am b/c the DPC liberty concept embraces the 1st Am guarantees. The Court has described the enforcement power as remedial. Legislation which alters the meaning of the free exercise clause cannot be said to be enforcing the clause. Congress does not enforce a Const right by changing what the right is. Congress has NO substantive, non-remedial power under the 14th Am. Policy: if Congress could define its own powers by altering the 14th Ams meaning, no longer would the Const be superior paramount law, unchangeable by ordinary means. It would be on a level w/ ordinary legislative acts, and like other acts, alterable when the legislature shall please to alter it. Slippery slope: what principle would possibly limit Congressional power? Shifting legislative minorities would constantly be changing the Const. When preventive rules like this one are enacted, there has to be a congruence b/w the means used and the ends to be achieved. RFRA is out of proportion to its goal. It appears to 26

attempt a substantive change in Const protections. The Court gets to interpret the Const, not Congress! (Marbury) Discussion: Before this, the enforcement power had been interpreted under a rational basis test. (If there was a rational basis to believe that the activity needed to be regulated to protect the rights in the 13-15 Am, was permissible.) 1990sthe Court saw a slippery slope and wanted to control this power. Sherbert (1963): set forth a balancing test if Congress passes a regulation burdening the free exercise of religion, it must submit to strict scrutiny Smith (1990): overruled Sherbert; you only get strict scrutiny when the govt discriminates against a particular religion (as opposed to placing an incidental burden on its exercise) -Then Congress passed RFRA in response, trying to restore freedom and go back to Sherbert, but this case said NO. Only the Court defines what 14th Am rights are. Congress cant make its own constitutional doctrine. In sum: the Court defines a right, which defines what constitutes a violation of that right. Congress can come up with remedies and try to prevent violations. BUT it must have a reasonable basis to believe that the activity its regulating has a connection to the violation of the right. The less congruent and proportional preventive rules are, the more they entail substantive effect. Congress on its own cannot define the right. The Court is the only institution allowed to interpret the Const. -Is this at odds with Art. I 8; McCulloch? Congress enacts the Religious Freedom Restoration Act under their 14th amendment powers, as a response to overrule Smith. 1st amendment granting freedom of religion is applicable to the states via the 14th amendment. Prohibits government from substantially burdening a persons exercise of religion unless the government can demonstrate the burden: (1) is in furtherance of a compelling governmental interest; and (2) is the least restrictive means of furthering that compelling governmental interest. Legislation which deters or remedies constitutional violations can fall within the sweep of Congress enforcement power even if in the process it prohibits conduct which is not itself unconstitutional [However,] Congresshas been given the power to enforce, not the power to determine what constitutes a constitutional provision. While the line between measures that remedy or prevent unconstitutional actions and measures that make a substantive change in the governing law is not easy to discernthe distinction exists and must be observed. RULE: Any legislation that regulates any activity that itself is not an unconstitutional activity must be Congruent and Proportional to remedying or preventing other unconstitutional activities. Must show the link between the non-prohibited constitutionally activity being regulated and the constitutional violations. There must be Congruence and Proportionality b/w the injury to be prevented or remedied; and the means adopted to that end. Otherwise legislation may become substantive in operation & effect. Congruence & Proportionality Test Show that statute is aimed at curing or preventing potential constitutional violations. Applicability of this test is vague. See Kimel v. Florida Board of Regents Congressional Power to Authorize Suits Against State Governments Article III 2 jurisdiction of the federal courts. Includes suits between a state and citizens of another state. State sovereignty was justified under this section such as the States could sue citizens of other states, but not the other way around. 27

11th Amendment The judicial power of the U.S. shall not be construed to extend to any suit in law or equity, commenced or prosecuted against One of the United States by citizens of another state, or by citizens or subjects of any foreign state. Hans v. Louisiana (1890) pg. 264 Expands 11th Amendment to cover lawsuits by citizens of the state (as well as noncitizens) arising under the constitution or laws of the U.S. The judicial power of the U.S. does not extend to suits by citizens against their own states. (States immunity from suit as a presupposition of the constitutional order) -This is blatantly at odds with the text of the 11th Am. (sovereign immunity of states against suits by citizens of other states) Qualifications: Ex Parte Young A federal court could issue an injunction against state officials acting in contravention of federal law. Fitzpatrick v. Bitzer Congress could abrogate the states 11th Amendment immunity and allow states to be sued for retrospective damages pursuant to its enforcement power under 5 of 14th Amendment if it does so. State can waive immunity. Basic Rule: Congress can only authorize suits against states pursuant only to 5 of the 14th Seminole Tribe of Florida v. Florida (1996) pg. 268 Congress does not have the power to authorize private suits for Money damages against the states under Article I powers (including commerce). Important distinguishing reasoned that because 11th amendment came after Article I limiting Congresss power in application to the states, but 14th came after 11th so it is not as limited by state immunity. Money damages are only allowed in actions against states in which the statute is enacted pursuant to 5 of the 14th Amendment. Alden v. Maine (1999) Congress does not have the power to authorize private suits for damages against the states under the Commerce Clause in State Court. Another case at odds w/test of 11th Am. Note that if Congress legislates under the 13/14/15 Am (etc.) it can authorize citizens to sue. Cases Denying Congressional Authority to Authorize Suits Against State Governments Florida Prepaid Postsecondary Edu. Expense Board v. College Savings Bank & US (1999) pg. 273 *Narrow Approach Case involves patent claim infringement. Florida infringed on CSBs patent. Patents are a matter of a federal law. Under Patent Remedy Act, state sovereignty is abrogated pursuant to the 14th Amendment (so that money damages can be claimed). Argument for 14th Amendment is deprivation of property (Due Process clause). Valid argument. Court rejects the claim because there was no record of widespread patent infringement amongst the state in which Congress intended to regulate. There is no congruence and proportionality to unconstitutional violations by the states. Kimel v. Florida Board of Regents (2000) pg. 278 *Narrow Approach Age Discrimination in Employment Act employers including a State. Valid under the Commerce power. But does not fall under 5 of the 14th amendment. No valid evidence of widespread Age discrimination amongst state employers. Therefore damages cannot be recovered.

28

Fails congruence & proportionality test because it prohibits substantial more state employment decisions & practices than would likely be held unconstitutional under the applicable equal protection, rational basis standard. Commentary: Federal Agencies can sue States on a persons behalf for damages. Board of Trustees, University of Alabama v. Garrett (2001) pg. 282 Title I of Americans with Disability Act. Cannot be enforced under 14th, but can under Commerce power. So states can be sued for injunctive relief, but not for monetary damages. United States v. Georgia (2006) pg. 302 Title II of the ADA creates a private cause of action for damages against the states for conduct that actually violates the 14th amendment, Title II validly abrogates state sovereign immunity. Valid abrogation because 8th amendment violations involved. Analysis: When can a State be sued for violating a federal law that authorizes such suits? 1. Constitutional Violation alleged? If Yes, State can be sued. If No, then is statute dealing with a type of discrimination that receives strict scrutiny or a fundamental right? o If Yes, State can be sued. o If No, the State can only be sued if Congress finds pervasive unconstitutional state conduct. Levels of Constitutional Scrutiny: 1. Rational Basis Review State regulation will be upheld if it is rationally related to achieving a legitimate state purpose. Applies primarily to social & economic regulations. (age & disability) 2. Intermediate Scrutiny law must be substantially related to achieving a substantial government purpose. (gender discrimination) 3. Strict Scrutiny Oregon v. Smith, City of Boerne v. Flores justified by a compelling government interest and is the least restrictive method. (racial discrimination) What is the relationship between forms of constitutional scrutiny and 5 powers of the 14th? Congress has more power to regulate States unconstitutional conduct subject to Strict Scrutiny under the 14th amendment (10th Amendment Challenges are likely to be Rejected).

29

THE FEDERAL EXECUTIVE POWER ARTICLE II


INHERENT PRESIDENTIAL POWER
Youngstown Sheet & Tube Co. v. Sawyer (1952) pg. 318 President Truman feared a labor dispute in the steel manufacturing industry would limit the supply of steel in America and put America in danger during wartime. Orders Secretary of Commerce to seize steel mills in order to continue manufacturing. 2 sources of presidential power Constitutional or Congressional authorization. Congress expressly rejected a statutory proposal that would allow governmental seizure in cases of emergency. Other statute on point but President did not act in accordance with it. Constitutional Authority Commander in Chief of Army & Navy, Power to execute laws of Congress. J. Jackson Concurrence relied on today. Commander in Chief response President has control over the theater of war, but the United States at the time is not a theater of war, so not acting in this capacity. Executing the laws response Not executing congressional policy, executing order directs presidential policy be executed. Acting as a lawmaker. Analysis pg. 321 Know for Exam 1. When the president acts pursuant to an express or implied authorization of Congress his authority is at its maximum, for it includes all he possesses in his own right plus all that Congress can delegate. 2. When the president acts in absence of either a congressional grant or denial of authoritythere is a zone of twilight in which he and congress may have concurrent authority In this area, any actual test of power is likely to depend on the imperatives of events 3. When the president takes measures incompatible with the expressed or implied will of Congress, his power is at its lowest ebb, for then he can rely only upon his own constitutional powers minus any constitutional powers of CongressCourts can sustain exclusive presidential control in such a case only by disabling the Congress from acting upon the subject. Presidents order falls within 3rd category. United States v. Nixon, President of the United States (1974) pg. 329 Facts: Employees of Nixons reelection committee broke into the DNC headquarters at the Watergate Hotel. The next year, a Senate Select Committee was set up by the Senate to investigate the affair. The committee was charged w/investigating the alleged illegal break-in and the question of White House involvement. Nixon appointed Cox as special prosecutor. Nixon was himself implicated and a member of the House introduced an impeachment resolution. The impeachment hearings were ongoing at the time of this decision. This case has to deal w/Nixons motion to quash a third-party subpoena that directed the President to produce certain tape recordings and documents relating to his conversations with advisers. Holding: The subpoena must be enforced. There is no unqualified Presidential privilege of immunity from judicial process. Reasoning: The exec has delegated its prosecutorial power to a special prosecutor, also within the exec branch, so the Pres. has given up his discretion on the matter. Also, ever since Marbury weve said its the Courts duty to say what the law is. (Issue #1) The interest in confidentiality itself is not enough. Only a claim of need to protect military, diplomatic, or sensitive national security secrets might be enough to give some immunity from judicial process. An unqualified privilege would impede the courts from doing justice. The privilege must be very limited only to those situations just 30

mentioned. Absent those situations, there must be a balance in every case: importance of general privilege of confidentiality in performance of Pres duties vs. the inroads of such a privilege on the administration of justice. (Issue #2) Here, were pretty sure the disclosure isnt going to cause major harm. But allowing the privilege would gravely impair court functioning and due process of law. Discussion: The key fact here may be that this was a subpoena issued by a JUDGE. The Court doesnt like the fact that the President wants to determine what his own privileges scope is. So the issue a clear rule in response. And, their decision uses balancing (non-rule approach) to decide in favor of the judicial branch. Watergate Scandal. Criminal matter. New Attorney General appoints a Special Prosecutor to handle the case. Finds out there are tape recordings regarding the investigation. Gets a subpoena for the tapes. President claims he is not subject to the subpoena because of Presidential Privilege. 1st issue is this a political question? Nixon argues it is because it is a dispute between officials in the executive branch. - No No judicial review of Presidential Privilege No. Court says what the law is. Even if not absolute, privilege prevails over subpoena not absolute, its qualified, based on circumstances. Information in a criminal matter is more important than privilege. Protected material has to be essential to the justice of the criminal proceeding. Rule: privilege based only on the generalized interest in confidentiality cannot prevail over the fundamental demands of due process of law in criminal proceedings. In 2004, Cheney v. U.S. Court affirms executive privilege when the matter is a civil action, rather than criminal. William J. Clinton, President v. City of New York (1998) pg. 335 Title XIX(19) of the Social Security Act - Gave huge sums of money to states to help pay for medical services for indigent people. Required those grants be reduced by the amount of certain taxes levied by the states on healthcare providers. Dept of Health and Human Services notified New York that 15 of its taxes were covered by the act. Statute required them to return $955 million to the federal govt. Congress resolves issue in favor of NY, by enacting Balanced Budget Act of 1997 4722(c). 4722(c) identifies that the disputed taxes are deemed permissible health care related taxes, and in compliance with the 1991 statute. President Clinton sends notice to the Senate and the House cancelling 4722(c). Would be preferential treatment, increased Medicaid costs, would have treated New York differently setting a costly precedent for other states to request the same. On the same day President cancelled provision granting limited tax benefits in the Taxpayer Relief Act, which was specifically subject to the line item veto in the Act. Line Item Veto Act Presidential power to cancel in whole 3 types of provisions signed into law 1. Any dollar amount of discretionary budget authority 2. Any item of new direct spending 3. Any limited tax benefit Cancellations can be reversed with a disapproval bill, making cancellations null and void. President cannot cancel disapproval bill but can veto it. No bill passed. Repeal of statutes must conform to Article I. No Constitutional powers given to the president to enact, amend, or repeal statutes. Line Item Veto Act unconstitutional. Any new procedure that changes the role of the President must be changed via Constitutional Amendment process, not Legislation.

31

Administrative Agencies Have legislative, executive, and judicial powers. Needed because Congress cannot handle the increasing # of minor concerns.

NON-DELEGATION DOCTRINE
Non-delegation doctrine Congress cannot delegate its powers to the President or executive administrative agencies without providing policy standards and guidance for the delegated powers. A.L.A. Schechter Poultry Corp. v. United States (1935) pg. 341 Congress cannot delegate its powers to the President. The National Recovery Act granted the President the powers to create a Code of Fair Competition. The act did not supply any standards for trade, industry, or activity. Delegation was unconstitutional. Panama Refining Co. v. Ryan (1935) pg. 343 There are limits of delegation, which there is no constitutional authority to transcend. Although the Constitution has never been regarded as denying to Congress the necessary resources of flexibility and practicality, Congress is forbidden to delegate the essential legislative functions which it is vested by Article I, Section:1 and Article I, Section:8 of the United States Constitution. Whitman v. American Trucking Assn., Inc. (2001) pg. 344 Synopsis of Rule of Law: When conferring decision making authority upon agencies, Congress must lay down an intelligible principle to which the person or body authorized to act is directed to conform. Holding: Under previous decisions, therefore, when Congress confers decision making authority upon agencies Congress must lay down by legislative act an intelligible principle to which the person or body authorized to [act] is directed to conform. Strictly interpreting the language of Section: 109(b)(1), the Supreme Court read the statute as requiring the EPA to set air quality standards at the level that is requisitethat is, not lower or higher than is necessaryto protect the public health with an adequate margin of safety, and so concluded that the scope provided by Congress for the EPA was well within the outer limits of our non-delegation precedents. Touby v. U.S. (1991) Congress does not violate the Constitution merely because it legislates in broad terms leaving discretion to executive or judicial actors, so long as Congress lies down, by legislative act, an intelligible principle.

LEGISLATIVE VETO
Immigration & Naturalization Service v. Jagdish Rai Chadha (1983) pg. 347 Facts: Chadha is a foreigner legally admitted to the U.S. in 1966 on a non-immigrant student visa. It expired in 1972. An immigration judge decided, as authorized by a statute, that Chadha should not be deported. Congress, under the statute, had power to veto the determination not to deport. So Congress passed a resolution opposing the granting of permanent residence to Chadha. Because the resolution was pursuant to the statute, it was not treated as an Article I legislative act, it was not submitted to the Senate or presented to the President for action. Holding: The Congressional veto provision is unconstitutional. Reasoning: Do the procedural requirements (bicameralism and presentment) of Art. I 7 apply? Whether actions taken by either House are, in law and fact, an exercise of legislative power depends not on their form but upon whether they contain matter which is properly to be regarded as legislative in character and effect. This resolution was essentially legislative in purpose and effect. Its purpose and effect was to alter the legal rights, duties, and relations of persons outside the legislative branch (Chadha, the Attorney General, exec branch officials). Without the provision in 32

the statute allowing such a resolution, this effect could have only been achieved through real legislation. Cumbersomeness and delays may not be avoided by skipping over explicit Const. standards. Concurrence: (Powell) Fear: the exercise of unchecked power. Congress, unlike the judiciary or agencies, is not bound by established substantive rules or procedural safeguards (e.g. right to counsel or a fair hearing). Congress exceeded its authority here. We dont need to decide whether all legislative vetoes are invalid under the presentment clauses. Dissent: (White) The legislative veto shouldnt be gotten rid of. Its important tool allowing the Pres and Congress to resolve major policy differences and preserve Congress control over lawmaking. Fears are not justified: the history of the legislative veto shows that Congress has not used it to aggrandize itself at the expense of the other branches. It just uses it to defend itself from the expanding power of the exec. Our federal govt needs flexibility to respond to contemporary needs w/o losing sight of fundamental democratic principles. FEAR: paralyzed, impotent govt. Discussion: The majoritys approach is similar to Blacks in Youngstown. Burger uses a bright line classification and rule: single house is exercising legislative power. This power requires bicameralism and presentment. Whites argument is more modernist, relating to the interdependence and complexity that led to more and more delegation to the agencies but also led to a need to check that delegated power. Immigration & Nationality Act required certain agencies to submit certain decisions for review. Either 1 or both houses could veto the action. Attorney General uses discretion delegated to him by Congress not to deport an alien. Unconstitutional because the resolution that would subject Chadha to deportation only has to be reviewed by one house. Art. I 7 any legislation has to be ratified by a majority vote of both houses then submitted for authorization by the President. 4 Exceptions to Bicameralism: There are four provisions in the Constitution, explicit and unambiguous, by which one House may act alone with the unreviewable force of law, not subject to the President's veto: (a) The House of Representatives alone was given the power to initiate impeachments. Art. I, 2, cl. 5; (b) The Senate alone was given the power to conduct trials following impeachment on charges initiated by the House and to convict following trial. Art. I, 3, cl. 6; (c) The Senate alone was given final unreviewable power to approve or to disapprove Presidential appointments. Art. II, 2, cl. 2; (d) The Senate alone was given unreviewable power to ratify treaties negotiated by the President. Art. II, 2, cl. 2.

CHECKING ADMINISTRATIVE AGENCIES


Morrison v. Olson (1988) pg. 356 Facts: An Act allows for appointment of an independent counsel to investigate and if appropriate, prosecute certain high-ranking govt officials for violations of federal criminal laws. Holding: The Act is valid. Its independent counsel provisions dont impermissibly interfere with the Presidents authority under Art. II. Reasoning: Unlike in Bowsher, the Act places the removal power in the hands of the exec branch. Our analysis shouldnt define rigid categories of officials who may or may not be removed at will by the Pres, we just need to make sure that Congress isnt interfering w/ the Press exercise of the exec power. The real question is whether the removal restrictions are of such a nature that they impede the Press ability to perform his const duty. Its a question of degree. Here, the Press need to control the exercise of the independent counsels discretion is not SO CENTRAL to the functioning of the exec branch as to require, as a matter of const law, that the counsel be terminable at will by the Pres. The exec through the Attorney General retains ample authority to terminate for good cause. 33

So, the limitation does not deprive the Pres of control over the ind counsel enough to interfere w/his const obligation to ensure the faithful execution of the laws. Weve never held that the Const requires the 3 branches to operate with absolute independence. Unlike Bowsher, Congress isnt trying to usurp exec branch functions. Also, theres no judicial usurpation of exec functions. Dissent: (Scalia) ALL exec power must be vested in the Pres. There is no real dispute that the functions performed by the independent counsel are executive. The Act deprives the Pres of exclusive control over that exec activity, as the majority acknowledges. Slippery slope: once we depart from the text of the Const, just where short of that do we stop? FEAR: impotent executive. The mini-executive that is the independent counsel is cut off from the political check that we use against prosecutorial abuse. Discussion: Prosecution is a paradigmatic exec function (enforcement of the law). The Pres could not control the independent counsel or affect his appointment/removal, but that was OK under Chadha and Bowsher. Rehnquist uses modernist arguments: the institutions cannot rigidly be kept separate. The question is one of fact and degree. (1988) SC upholds the Independent Counsel (IC) position even though the IC is appointed by AG, cannot be removed by the Pres. (only by AG for cause or judiciary when investigation is done). Title VI of the Ethics in Government allows for appointment by a special division (court created by the act) of independent counsel to investigate and prosecute high-ranking government officials. Art. 2 Sec. 2 High-ranking officers must be appointed by the President and Approved by Congress. Inferior officers can be appointed by the President, the Courts, or heads of departments when Congress gives them the power via statute. (Congress cannot vest in itself the appointment power). IC is an inferior officer (not principal) because of IC's subordination to the Attorney General so the appointment is OK. Not a problem of incongruous delegation of function - because the courts have a special expertise in the area of appointing prosecutors, and there are problems with having the executive investigate itself. Issue: For cause requirement restricts WHEN the Pres. can remove an executive officials Limiting Pres. Constitutional Power to conduct criminal investigations. No SOP problem - because CG doesn't have removal power or any other control over IC - the for cause requirement does not really infringe on PR's executive power to investigate crimes. No SOP problem of CG aggrandizing - it doesn't gain power at the expense of PR. Nor does judiciary really get power. Congress is not taking power for itself at the expense of another branch. Dissent: Art. 2 gives PR all executive power. Investigations belong to the executive branch. This gives CG the power to severely harass executive. Point of the statute is the reason for invalidating it. Isolating IC from the officials they are investigating. The primary check against prosecutorial abuse is a political one. Position of IC itself is unconstitutional because it usurps Pres. Power to control criminal investigations. Yes the idea of executive investigating itself is scary - but no more so than CG passing laws about it's salary or the judiciary judging criminal investigations of judges. This gives CG to create any number of executive type positions but to retain removal power and take it away from PR. Appointment Congress may not vest appointment power of high-ranking Executive offices within itself, but can vest appointment of inferior offices in itself. pg. 360-361 Removal The President may remove executive branch officials unless removal is limited by statute.

34

PRESIDENTIAL WAR POWERS *Constant struggle over war powers between President and Congress. Courts try to stay out of argument.
Hamdi v. Rumsfeld pg. 382 Facts: Hamdi, an American citizen, was seized in Afghanistan and turned over to the U.S. military. He was transferred to a military prison in South Carolina. The govt contends he is an enemy combatant and that this status justifies holding him in the U.S. indefinitely w/o formal charges or proceedings, unless and until it makes the determination that access to counsel or further process is warranted. Hamdi claims a violation of due process, and that 4001(a) states no citizen shall be imprisoned or otherwise detained by the United States except pursuant to an Act of Congress. However there is an Act of CG AUMF gives Pres. power to use necessary and appropriate force against enemy combatants. Holding: Although Congress authorized the detention of combatants in the narrow circumstances alleged here, due process demands that a citizen held in the U.S. as an enemy combatant be given a meaningful opportunity to contest the factual basis for that detention before a neutral decision maker. Reasoning: Congress authorized Hamdis detention through the AUMF (Authorization of Use of Military Force Resolution). That law is explicit congressional authorization for the detention of individuals in this narrow category (enemy combatant). This detention is so fundamental and accepted an incident to war as to be an exercise of the necessary and appropriate force Congress authorized the Pres. to use. Detention can be for the duration of the relevant conflict. Hamdi has not made any concessions eliminating the right to further process: just b/c youre captured in a zone of active combat doesnt mean youre necessarily an enemy combatant. We have to balance values a citizen-detainee seeking to challenge his classification as an enemy combatant must receive notice of the factual basis for his classification, and a fair opportunity to rebut the govts factual assertions before a neutral decision maker. At the same time, the exigencies of the circumstances may demand that aside from these core elements, enemy combatant proceedings may be tailored to alleviate their uncommon potential to burden the exec in a time of conflict. The govt is wrong in saying that SOP principles mandate a heavily circumscribed role for the courts in these circs. We have long made it clear that a state of war is not a blank check for the Pres. when it comes to citizens rights (Youngstown). Concurrence: (Souter) The AUMF does not authorize Hamdis current detention based on the evidence the govt has come forward with. We require Congress to give a clear statement of authorization to detain as a check on the exec. However, in a time of genuine emergency, exec may be able to detain a citizen if there is reason to fear he is an imminent threat. But there is no emergency. Says nothing about detention in AUMF. Dissent: (Scalia) There has been no suspension of habeas corpus. The executives assertion of military exigency has never been thought sufficient to permit detention w/o charge. If civil rights are to be curtailed during wartime, it must be done openly and democratically. Has to be tried for treason or some other crime. (Thomas) The detention falls squarely within the federal govts war powers, and we lack the expertise and capacity to second-guess that decision. The habeas challenge should fail. The plurality fails to account for the govts compelling interests and for our own institutional inability to weigh competing concerns. *Point of section 4001(a) was not to allow the Pres. to use his power to detain American citizens without authorization from Congress Johnson v. Eisentrager (1950) pg. 397 21 Germans convicted of violating laws of war- fighting after surrender. Detained in China; put to military trial in China by US authorities. Only US involved in trial. Deported to Germany to serve sentences. Under guard of an American in a German prison. Prisoner filed habeas corpus petition in 35

US- unlawful conviction. SCOTUS denies- no instance where the writ has been issued to an alien enemy when that enemy has never been under US jurisdiction. This case formed basis of Bushs Guantanamo policy Take all the people that we capture in the war on terror and ship them to Gitmo, then claim they have no recourse in American court. Why Hamdi was shipped to US- he was a citizen. Rasul v. Bush (2004) (statutory right to habeas corpus) Decided same day as Hamdi. Habeas petitions by non-citizens, captured abroad, held in Gitmo. Lower courts rely on Eisentrager. SCOTUS- application of habeas statute (28 USC 2241) to persons at Gitmo is consistent w/ historical reach of the writ of habeas corpus. Could file petition under statute Congress enacts DTA (2005) to eliminate statutory right of habeas corpus (overrule Rasul). Also contains a process to determine combatant status CSRT (combatant status review tribunals)- no full trial, no right to counsel, presumption that evidence against you is correct, no availability to present evidence on your behalf, hearsay is admissible Hamdan v. Rumsfeld (2006) (review of DTA) DTA does not strip federal court jurisdiction over cases pending on the day of DTAs enactment Rules specified for Hamdans trial are in violation of UCMJ Procedure Pres set up was in violation of federal statute Congress then enacted MCA (2006) to do two things: 1) DTA applies to all cases, even those pending- no gitmo detainees have habeas corpus right & 2) Gave Pres stat authority to construct the commissions (trials) in the same way that was prohibited by the UCMJ

SUSPENSION CLAUSE
Boumediene v. Bush (2008) pg. 393 Constitution habeas corpus right or a Statutory habeas corpus right. Detainees outside of the US have no habeas corpus right unless by statute, which there was at one point but Congress revised statutory law to revoke those rights. Detainees at Guantanamo file writ for habeas corpus. Suspension of Habeas Corpus Article I 9 Johnson v. Eisentrager Held: Access to American courts does not extend to foreigners in military custody who had no presence in any territory over which the U.S. is sovereign. 3 factors for determining reach of the Suspension Clause and that distinguish Boumediene from Eisentrager: - pg. 399-400 1. The citizenship and the status of the detainee and the adequacy of the process through which that status determination was made; Non-citizens like in Eisentrager Status lawful vs. unlawful enemy combatant. In Eisentrager they admitted to being enemy combatants (but denied being unlawful enemy combatants) Adequacy Eisentrager s received a full military trial. 2. The nature of the sites where apprehension and then detention took place; and U.S. has de facto sovereignty of Guantanamo Bay (not official sovereignty). Not the case in Eisentrager, U.S. had no control over Landsberg Prison in Germany. 3. The practical obstacles inherent in resolving the prisoners entitlement to the writ. Easier to transport prisoners from Guantanamo to U.S. than from a warzone in Germany.

36

Military Commission Act of 2006 violates the Suspension Clause of Article I Section 9. Detainee Treatment Act of 2005 statute cannot be construed to allow the Constitutionally required opportunity for the detainee to present relevant exculpatory evidence that was not made part of the record in the earlier proceedings. Dissent: (Scalia) Writ of habeas corpus does not and has never extended to aliens abroad. DTA sufficient.

MILITARY TRIBUNALS
Ex Parte Quirin (1942) pg. 412 German Nazi soldiers (1 American citizen) detained in New York and Chicago. Claim President is without authority to establish the Military Commissions that tried them. Court Congress has authorized Military Commissions so the President as Commander in Chief has the constitutional power. Unlawful combatants are subject to trial and punishment by tribunals for acts, which render their belligerency unlawful. Citizenship in the U.S. of an enemy belligerent does not relieve him from these consequences. Therefore, because the amendments cannot be read "as either abolishing all trials by military tribunals, save those of the personnel of our own armed forces, or, what in effect comes to the same thing, as imposing on all such tribunals the necessity of proceeding against unlawful enemy belligerents only on presentment and trial by jury," the rights of the conspirators were not violated.

PRESIDENTIAL IMMUNITY
Nixon v. Fitzgerald (1982) pg. 420 Analyst for the Air Force was fired by Nixon after he testified about embarrassing matters in front of Congress concerning the Department of Defense. Federal Whistle Blower statute. The Press duties are of singular importance. Diversion of his energies by concern w/private lawsuits would raise unique risks to the effective functioning of govt. B/c the Pres is so prominent, he would be an easily identifiable target for suits for civil damages. This would distract the Pres from his public duties. FEAR of accountability-free President is allayed by the fact that we have other safeguards: impeachment, oversight by Congress, desire to earn re-election, etc. POTUS is entitled to absolute immunity from damages liability predicated on his official acts. But the question of whether POTUS has acted within the scope of his constitutional and statutory duties is not covered by absolute immunity. POTUS is NOT immune from criminal liability. POTUS is immune from civil suits pertaining to his official act. Do not want POTUS to be unduly cautious in performing his duties. Litigation would divert his time from more important matters. Clinton v. Jones pg. 423 Facts: Jones alleged that in 1991, while Clinton was governor of Arkansas, he made sexual advances toward her, violating federal and state law. She sought monetary damages. Clinton argued that litigation should be delayed until the conclusion of his presidential term. Holding: The litigation will not be delayed. Reasoning: We can distinguish Fitzgerald b/c that case involved official conduct. Immunity in the official conduct context serves the public interest by enabling officials to perform their designated functions effectively w/o fear of personal liability. But the presidential immunity does not extend beyond the scope of action take in an official capacity. This decision will in no way curtail the powers of the exec branch (no SOP problem). The litigation of questions re: unofficial conduct poses no perceptible risk of misallocation of judicial or executive power. History suggests that the number of cases filed wont be overly large.

37

Discussion: Court here not worried about chilling of Pres. duties, similar to Nixon (whereas were worried about this chilling in Fitzgerald. POTUS could be sued while holding office for actions before holding offices. Nor must the Court have to stay the litigation until after he leaves office. Immunity is inapplicable to unofficial conduct.

IMPEACHMENT
Pg. 427 Article 2 4 Only removed from office when impeached for treason, bribery, High crimes, or misdemeanors. Article 1 2 House of Representatives have sole power of impeachment. Article 1 3 Senate has sole power to try all impeachments. Cannot be convicted without concurrence of 2/3 votes.

38

LIMITS ON STATE REGULATORY & TAXING POWER


PREEMPTION
When states regulatory powers are invalidated: When Congress is silent, yet dormant commerce clause may invalidate power. When Congress has preempted states power via legislation. Preemption: Express preemption Federal statute specifically states it preempts the states from regulating in that certain area. Implied preemption o Field Preemption: Congress meant to occupy the field and displace any state regulation. o Conflict preemption: Where a state regulation conflicts with or is irreconcilable with or is inconsistent with the federal legislation. (Hines v. Davidowitz). o Impediment to the full attainment of federal purpose: Where a state regulation curtails or frustrates the full purpose of the federal regulation. Chamber of Commerce of U.S. v. Whiting (2011) supp. pg. 49 Legal Arizona Workers Act (1) allows superior courts in Arizona to suspend or revoke business licenses of employers who knowingly or intentionally hire unauthorized aliens and (2) also makes participation in E-Verify (a system that determines eligibility for employment based on information from I-9 forms and U.S. Department of Homeland Security and Social Security Administration records) mandatory for all employers. (1) Immigration Reform & Control Act (2) Preemption: The provisions of this section preempt any state or local law imposing civil or criminal sanctions (other than through licensing and similar laws) upon those who employ, or recruit or refer for a fee for employment, unauthorized aliens. Express & Implied Preemption Claims. If there is a difference between federal licensing and state licensing, then only Expressed preemption remains. Expressed Preemption Claim There is a difference between Arizona licensing and federal licensing so Arizona licensing law falls out of the preemption exception. Court Rejects Implied Preemption Claim Arizona laws frustrate the purpose of the IRCA. Purpose of the federal statute was to protect legal aliens from wrongful determinations, and to protect employers from wrongful determinations they are employing illegal aliens. Court Rejects (2) Immigration Responsibility Act The Secretary of Homeland Security shall provide for the operation of the E-verify Program Voluntary election any person or other entity that conducts any hiringmay elect to participate in that pilot program, The Secretary of Homeland security may not require any person or other entity to participate in a pilot program. Implied Preemption Claim Not Conflict because can adhere to both laws at the same time. Impediment of the full purpose of the statute. Purpose of IRA was for it to be voluntary so that employers would have options in finding out whether a potential employee is an illegal alien. Court Rejects Wyeth v. Levine (2009) supp. pg. 58 If a drug meets the labeling requirements of the FDA, does that give rise to federal preemption of state law regarding inadequate labeling? Wyeth presented two arguments in favor of FDA preemption: 1. It is impossible for Wyeth to comply with both the state-law duties and federal labeling regulations, since the latter forbids it from changing its label without FDA approval. 39

2. Permitting states to require stronger warnings creates an unacceptable "obstacle to the accomplishment and execution of the full purposes and objectives of Congress," Hines v. Davidowitz, because it substitutes a lay jury's decision about drug labeling for the expert judgment that Congress sought to entrust with drug labeling decisions when it created the FDA. Questions of federal preemption "must be guided by two principles of our preemption jurisprudence": 1. The purpose of Congress is the ultimate standard in every preemption case. 2. In all preemption cases, and particularly in those in which Congress has legislated in a field, which the States have traditionally occupied, we start with the assumption that the historic police powers of the States were not to be superseded by the Federal Act unless that was the clear and manifest purpose of Congress. 1st argument, Wyeth is incorrect that relabeling the drug to conform to Vermont law would necessarily have violated federal labeling regulations. Although a manufacturer generally needs FDA approval before changing a drug label, the agency's "changes being effected" (CBE) regulation permits certain unilateral labeling changes that improve drug safety. Wyeth's misreading of this regulation is based on the misunderstanding that the FDA, rather than the manufacturer, bears primary responsibility for drug labeling. It is a central premise of the Food, Drug, and Cosmetic Act (FDCA) and the FDA's regulations that the manufacturer bears responsibility for the content of its label at all times. Pg. 1116. 2nd argument, Wyeth is incorrect that permitting states to require stronger warnings would interfere with Congress' purpose of entrusting an expert agency with drug labeling decisions because it was not Congress's intent, in writing the Food, Drug, and Cosmetic Act, to preempt state-law failure to warn actions. Wyeth's argument misconstrues the intent of congress behind the FDCA. Congress did not intend the FDCA to preempt state-law failure-to-warn actions. Wyeth's argument also misconstrues the capacity of agencies to preempt state law, as Wyeth's argument relies on the preamble to a 2006 FDA regulation declaring that state-law failure-to-warn claims threaten the FDA's statutorily prescribed role. Although an agency regulation with the force of law can pre-empt conflicting state requirements, this case involves no such regulation but merely an agency's assertion that state law is an obstacle to achieving its statutory objectives. Where, as here, Congress has not authorized a federal agency to preempt state law directly, the weight this Court accords the agency's explanation of state law's impact on the federal scheme depends on its thoroughness, consistency, and persuasiveness. Under this standard, the FDA's 2006 preamble does not merit deference: It is inherently suspect in light of the FDA's failure to offer interested parties notice or opportunity for comment on the pre-emption question; it is at odds with the available evidence of Congress' purposes; and it reverses the FDA's own longstanding position that state law is a complementary form of drug regulation without providing a reasoned explanation. No Preemption because the State law furthers the federal statute if not helps it. Hines v. Davidowitz (1941) pg. 448 Where the federal government has enacted a complete scheme of regulation and has therein provided a standard for registration of aliens, states cannot inconsistently with the purpose of Congress, conflict or interfere with, curtail or complement, the federal law, or enforce additional or auxiliary regulations, Congress has provided a standard for alien registration in a single integrated and allembracing system Field Preemption. Analysis waivers from field to conflict preemption.

40

DORMANT COMMERCE CLAUSE


Dormant Commerce Clause some statutes invalid even without Congressional action because they unduly burden interstate commerce. 3 Modern Categories of Dormant Commerce Clause Invalidations 1. State laws that facially discriminate against out-of-state commerce. 2. Facially neutral state laws whose purpose [or effect] is to discriminate against out-of-state commerce. 3. Facially neutral state laws whose purpose is NOT to discriminate against out of state commerce but have a disproportionate adverse effect on interstate commerce. Scrutiny for Categories 1&2 State must show that the law both serves a legitimate local purpose and that this purpose cannot be achieved by available nondiscriminatory means. (Maine v. Taylor) Lower Scrutiny for Category 3 Balancing Interest. *Argument against Dormant Commerce Clause Constitution only grants power to Congress to regulate interstate commerce, does not take power away from the States to regulate when Congress has failed to do so. Judiciary is taking upon itself the power to invalidate state legislation. City of Philadelphia v. New Jersey (1978) pg. 464 NJ enacted statute that prohibited the importation of solid or liquid waste, which was collected or originated in another state. Economically Protectionist measure to protect the in-state customers of landfills vs. Environmental protectionist measure. Court Purpose doesnt matter because the act still discriminates against articles of commerce coming from outside the state, unless there is some reason, apart from their origin, to treat them different. Statute falls under 1st category because it is explicit in the language of the statute presumption of unconstitutionality. Maine v. Taylor (1986) pg. 490 Facially discriminatory statute. But constitutional because the importation of baitfish would potentially endanger native fish. *Hunt v. Washington Apple Advertising Commission. (1977) pg. 480 North Carolina enacts a statute which required all closed containers of apples sold, offered for sale, or shipped into the state to bear no grade other than the applicable U.S. grade or standard. State grades were expressly prohibited. Category 2. Effect is Discriminatory North Carolina is preventing WAAC from using their competitive advantage, while shielding local producers from outside competition. Dean Milk Co. v. City of Madison, Wisconsin (1951) pg. 489 Any milk sold in the city had to be pasteurized and bottled at a plant within 5 miles of the city. Claimed it was to protect the health of its citizens. Unconstitutional. 3rd Category of Dormant Commerce Clause Invalidations Pike v. Bruce Church Inc. (1970) pg. 493 Statute does not have a discriminatory purpose. Effectuates a legitimate local interest, and its effects on commerce are only incidental. Must be upheld unless the burden imposed on interstate commerce is clearly excessive in relation to the putative local benefits.

41

Consolidated Freight Corp. of Delaware v. Kassel (1981) pg. 497 Statute restricted the max length of vehicles. For commercial trucks carrying double loads, they would have to drive around state or drop a trailer before crossing the state. Iowa claims purpose is for driver safety. Court thinks the safety benefits are not increased because if there are more trucks carrying single loads across the state. The burden on interstate commerce outweighs the safety benefits. Statute unconstitutional. Bendix Autolite Corp. v. Midwesco Enterprises, Inc., 486 U.S. 888 (1988) (Scalia, concurring) Would not balance laws under category 3, should be upheld until Congress acts. Exceptions to Dormant Commerce Power pg. 501 Congressional Permission Congress can allow a state or local government to enact laws that are facially discriminatory White v. Mass. Council of Constr. Employers Market Participation When a state acts a market participant it can discriminate against out of state commerce. Fairness Doesnt create the dangers as discriminatory regulation South-Central Timber Development, Inc. v. Wunnicke (1984) pg. 506 Alaska selling state owned timber required that it be processed in the State. Alaska went beyond the bounds of Market participation to regulation. Argued that even if it did, that is constitutional under the 3rd category test. Court rejects saying it is facially discriminatory thus Category 1. Rule: If a state imposes burdens on commerce within a market in which it is a participant, but those burdens have a substantial regulatory effect out side of that particular market, they are invalid per se under Commerce Clause. Article IV 2 cl. 3 also provides protection to interstate discrimination but is limited.

42

CONSTITUTIONAL PROTECTION OF CIVIL RIGHTS & LIBERTIES


APPLICATION OF THE BILL OF RIGHTS TO THE STATES
Barron v. Mayor and City Council of Baltimore (1833) pg. 525 Facts: Barron sued the city for ruining his wharf in Baltimore harbor. He claimed that municipal street construction had diverted the flow of streams so that they deposited silt in front of his wharf, and that this made the water too shallow for most vessels. He argued this action violated the 5th Amendment Takings Clause. deprivation of property without Due Process. Holding: The just compensation provision in the 5th Amendment is intended only as a limitation on the exercise of power by the national govt and does not apply to state legislation. Reasoning: Fear: the abuse of power, the encroachment of the national government. Thats why the amendments were enacted. The amendments dont contain expressions indicating an intent to apply them to the state govt, so the court cant do that. The fear doesnt apply to the states. Also, textual argument that because elsewhere in the Const there are limits on state govt, so if framers of amendment had wanted to limit state govt, would have said so in the amendment. Discussion: Marshall echoes the view that citizen control over state govt serves as an adequate safeguard of individual liberty. The bill of rights, on the other hand, was needed b/c of special concerns about the power of the remote federal govt. The question thus presented, is, we think, of great importance, but not of much difficulty. The 5th amendment must be understood as restraining the power of the federal government, not as applicable to the states. The states are today restrained by almost all of the Bill of Rights via DPC The Civil Rights Amendments explicitly required state adherence Amendment 14 No State shall make or enforce any law, which shall abridge the privileges or immunities of citizens of the United States; nor shall any state deprive any person of life, liberty, or property, without due process of law; nor deny to any person within its jurisdiction the equal protection of the laws. The Slaughterhouse Cases (1872) pg. 528 Review this! Facts: Louisiana legislature passed a statute granting to a slaughterhouse company the exclusive right to engage in the livestock landing and slaughterhouse business w/in an area including the city of New Orleans. The company was required to permit any person to slaughter animals in its slaughterhouse at charges fixed by law. Plaintiffs, butchers whose businesses were restricted by the statute, sued to invalidate the monopoly under the new 14th Amendment. Holding: The privileges/immunities plaintiffs rely on are privileges/immunities of state citizenship, and thus are not protected by the federal Constitution. Reasoning: The Court interprets the new 14th Amendment very narrowly. Fundamentalist argument: it has a unity of purpose and thus a true meaning. That purpose is the freedom of the slave race and the protection of the newly freed slaves. Citizenship in a state is different from U.S. citizenship. Different rights are guaranteed. Only the privileges/immunities of U.S. citizenship are protected by the Const; the privileges/immunities of state citizenship are protected by the democratic process. We dont want to fetter and degrade the state govt by subjecting them to the control of Congress when theyre dealing with the rights that are universally conceded to them. The Court declines to define what the privileges/immunities of U.S. citizenship are at this time. Dissent: (Field) The privileges/immunities of all U.S. citizens are secured against abridgment in any form by any state. (Bradley) The state has broad authority to regulate its citizens conduct, but there

43

are certain fundamental rights, which this right of regulation cannot infringe. (This was the purpose of the amendments.) Fears of interference by the national govt are unwarranted. Discussion: The Court demonstrates its faith in state political processes. Fear: national govt (powerful and ruthless during Reconstruction); the court as censor (judicial tyranny) need to draw a boundary. There was nothing necessary about the textual interpretation/distinction the Court came up with. It was cooked up out of fear. This is a bright line rule. This court is called upon for the first time to give construction to the 13th and 14th amendments. The most cursory glance (discloses) a unity of purpose, when taken in connection with the history of the times, which cannot fail to have an important bearing ontheir true meaning. The overshadowing and efficient cause [of the war of the rebellion] was African slavery. How do the slaughterhouse-cases treat the privileges and immunities clause? The citizens of the United States - Here there is a distinction to be made between the P&I of the citizens of the US and not of the States The clause the plaintiffs rely most upon speaksof privileges and immunities of the citizen of the USOf the privileges and immunities of the citizen of the United States, and of the Privileges and Immunities of the citizen of the State, and what they respectively are, we will not There are 2 types of Privileges & Imunnities: The P&I that arise from being a citizen of the U.S.; and the P&I that arise from being the citizen of a state. According to this case the most important of such rights are those that arise from your citizenship of a state. The one time where the court went against Slaughter-House with regard to the P&I clause was in Saenz v. Roe Saenz v. Roe (1999) pg. 534 Facts: When CA discriminated against citizens who had resided in the state for less than one year in distributing welfare benefits, the state statute was challenged by Saenz, alleging that the right to travel. CA argued that the statute should be upheld if there is a rational basis, and the states legitimate interest in saving over $10 million a year satisfied that test. Issue: Do durational residency requirements violate the fundamental right to travel by denying a newly arrived citizen the same privileges and immunities enjoyed by other citizens in the same state, and are they thus subject to strict liability? Holding: Yes. The court invalidated the statute under the P&I Clause of the 14th Amendment. Justice Stevens, writing for the majority, found that although the "right to travel" was not explicitly mentioned in the Constitutions, the concept was "firmly embedded in our jurisprudence" a fundamental right. He described three components of the right to travel: (1) The right to enter one state & leave another; (2)The right to be treated as a welcome visitor rather than a hostile stranger; (3) For those who want to become permanent residents, the right to be treated equally to native born citizens. The right of a newly arrived citizen to the same privileges and immunities enjoyed by other citizens of the same state is protected by the new arrivals status as a state citizen as well as her status as a citizen of the United States. Standard of Review: Strict Scrutiny Rule: Durational residency requirements violate the fundamental right to travel by denying a newly arrived citizen the same privileges and immunities enjoyed by other citizens in the same state and are therefore subject to strict liability. (Right to travel has long been protected by the EPC and DPC as well) By approaching this under the P&I clause, then it does not need to be a fundamental right that you are talking about. Dissent (Thomas): We shouldnt do this, because its not what was intended when the 14th Amendment was ratified. 44

Dissent (Rehnquist): Its okay to require people to reside in a state for a certain amount of time before they can terminate their marriage or vote in primary elections, so why not welfare? Substantive Due Process Allows free willing judicial inquiry. (Incorporation) Procedural Due Process guaranteed procedural rights; notice, service, etc.

INCORPORATION OF THE BILL OF RIGHTS INTO DUE PROCESS CLAUSE


*So far as a constitutional right is incorporated by the Due Process Clause, SO TOO ARE THE RIGHTS ASSOCIATED WITH THE CONSTITUTIONAL RIGHT, with a few exceptions. Twining v. New Jersey (1908) pg. 536 It is possible that some of the personal rights safeguarded by the first eight amendments against National action may also be safeguarded against state action, because a denial of them would be a denial a denial of due process law. If this is so, it is not because those rights are enumerated in the first eight amendments, but because they are of such a nature that they are included in the conception of due process. Test of whether a constitutional right is incorporated under the 14th amendment - We inquire whether the [exemption from self-incrimination] is of such a nature that it must be included in the conception of due process. Is it a fundamental principle of liberty and justice, which inheres in the very idea of free government, AND is the inalienable right of a citizen of such govt. If so and it is of a nature that pertains to process of law, it is essential to due process of law. Palko v. Connecticut (1937) pg. 539 Facts: A Connecticut statute permitted the state to appeal in criminal cases. Holding: The 14th Am does NOT incorporate the privilege against Double Jeopardy. Reasoning: Some rights have been incorporated (freedom of speech/press/assembly/religion, right to counsel), but not the 5th Am. right to be free from self-incrimination or the right to trial by jury. Those latter rights are important, but they are not fundamental (the very essence of a scheme of ordered liberty). To abolish them is not to violate a principle of justice so rooted in the tradition and conscience of our people as to be ranked as fundamental. Discussion: This case is important for arguing how we decide whether a right is incorporated: we ask whether the right is fundamental. Also the case acknowledges that due process can embrace not just procedural rights but also substantive rights like freedom of speech. The question is whether a right is of the very essence of a scheme of ordered liberty [so that as to abolish it] to violate a principle of justice so rooted in the traditions and conscience of our people as to be ranked as fundamental. This is true for illustration, of freedom of thought, and speech [But not of the prohibition on double jeopardy]. Adamson v. California (1947) pg. 540 Facts: In a state court prosecution, the prosecution was permitted to comment upon on a defendants failure to take the stand. Such a comment would infringe defendants privilege against selfincrimination if the trial were in a federal court. Holding: The 14th Am does NOT incorporate the privilege against self-incrimination. Dissent: (Black) P&IC incorporation The 14th Am was intended to make the Bill of Rights applicable to the states. We should follow that original purpose and give everyone the complete protection of the Bill of Rights. DPC incorporation This decision reasserts a constitutional theory that this Court is endowed by the Constitution with boundless power to expand and

45

contract constitutional standards to conform to the Courts conception of what at a particular time constitutes civilized decency and fundamental liberty and justice. Concurrence: (Frankfurter) Attacks Blacks theory of total incorporation. Judicial review of the due process guarantee inescapably imposes upon the Court an exercise of judgment upon the whole course of the proceedings in order to ascertain whether they offend the canons of decency and fairness, which express the notions of justice of our people. But the due process clause does not imply that the judges are wholly at large. Judicial judgment must move within limits of accepted notions of justice. To that end we should give some deference to the judgment of the state court under review. Duncan v. Louisiana (1968) pg. 543 Synopsis of Rule of Law. The Fourteenth Amendment of the Constitution guarantees a right of trial by jury in all criminal cases. Facts. The Appellant stated that he merely touched the boy on the elbow. The Appellant was charged with simple battery and requested a trial by jury. His request was denied. The trial judge concluded the elements of simple battery were proven by the state and found the Appellant guilty of the crime. Issue. Does a state law granting a jury trial only in cases where the penalty is capital punishment or imprisonment at hard labor violate the Constitution? Held. Yes. The Constitution was violated when Appellants demand for jury trial was refused. Trial by jury in criminal cases is fundamental to the American scheme of justice because it works to prevent governmental oppression. Right of trial by jury in serious criminal cases works as a defense against arbitrary law enforcement and qualifies for protection under the Due Process clause of the Fourteenth Amendment of the constitution. The purpose of a right to jury trial is to reduce the possibility of judicial or prosecutorial unfairness. Discussion. Trial by jury is an integral part of the criminal system. It is necessary to protect against unfounded criminal charges and against judges who are too responsive to higher authority. Trial by jury is also a safeguard against overzealous prosecutors. 5th & 6th Amendment Incorporation Test 1. Is the right among those fundamental principles of liberty & justice which lie at the base of all our civil and political institutions; 2. Whether it is basic in our system of jurisprudence; and 3. Whether it is a fundamental right essential to a fair trial. McDonald v. City of Chicago (2010) supp. pg. 69 2nd Amendment Right to Bear Arms Application to the States Incorporated by the 14th DPC. Chicago ordinance banned possession of handguns, even in the home. Test In answering [whether the 2nd amendment is incorporated in the due process clause], we must decide whether the right to keep and bear arms is fundamental to our scheme of ordered liberty, or as we have said in a related context, whether this right is deeply rooted in this Nations history and tradition. Application Self-defense is a basic right, recognized by many legal systems from ancient times to the present day, and in Heller, we held that individual self-defense is the central component of the 2nd Amendment right. Heller makes it clear that this right is deeply rooted in this Nations history and tradition. Thomas Concurrence 14th Amendment P&IC Incorporation. Stevens Dissenting - Even accepting the Courts holding in Heller, it remains possible that only part of the right isenforceable [against the states]. Conservatives Prefer P&IC as a vehicle for incorporations, because rights like abortion are substantive due process rights. Want to revisit Slaughterhouse Cases. Scalia disagrees with this 46

approach saying basically it is too little too late to use P&I incorporation because Due Process is established. Commentary: Total incorporation of the Bill of Rights has never been accepted by SCOTUS, However the Court has found almost all of the rights granted by the Bill of Rights to be incorporated via the Due Process Clause. For most part, all incorporated Bill of Rights provisions apply the same to the states as they do to the Federal government. Bill of Rights Provisions that have NOT been Incorporated: pg. 545 1. 3rd Amendment right to not have soldiers quartered in a persons home 2. 5th Amendment right to a grand jury indictment in criminal cases 3. 7th Amendment right to a jury trial in civil cases 4. 8th Amendment prohibition of excessive fines State Action Doctrine Many 14th Amendment (Constitutional) rights are only applicable to the government, not to private individuals. Exceptions pg. 552 Public Functions Exception a private entity must comply with the Constitution if it is performing a task that has been traditionally, exclusively done by the government. Entanglement Exception private conduct must comply with Constitution if the government has authorized, encouraged, or facilitated the unconstitutional conduct. Civil Rights Cases: U.S. v. Stanley pg. 548 Discussion. The Fourteenth Amendment of the Constitution prohibits only State action. The Fourteenth Amendment of the Constitution gave Congress the power to enforce the Amendment through appropriate legislation. The Fourteenth Amendment of the Constitution does not give Congress the power to regulate private rights.

47

ECONOMIC LIBERTIES
Economic Liberties generally refer to constitutional rights concerning the ability to enter into and enforce contracts; to pursue a trade or profession; and to acquire, possess, and convey property. Substantive Due Process Protecting Economic Liberties Rational Basis Review Safeguarding Privacy

LOCHNER ERA
Allgeyer v. Louisiana (1897) pg. 607 First rejection of a state law for violating economic substantive due process. Question: Does the Louisiana law violate the Fourteenth Amendment's due process clause which, according to Allgeyer and Company, protects its liberty to enter into contracts with businesses of its choice? Conclusion: Yes. The Court found that the Louisiana statute deprived Allgeyer and Company of its liberty without due process of law as protected by the Constitution's Fourteenth Amendment. The Court reasoned that even though the Atlantic Mutual Insurance Company of New York did not maintain an office or agent in Louisiana, Allgeyer and Company could still, as it did, enter into a contract with Atlantic Mutual in the state of New York to insure its Louisiana property. Lochner v. New York (1905) pg. 608 Facts: A New York statute provided that no employee shall work in a biscuit, bread or cake bakery or confectionary establishment more than sixty hours in any one week, or more than ten hours in any one day. Its constitutionality was challenged. Holding: An act must have a direct relationship, as a means to an end, and the end itself must be appropriate and legitimate, before an act can be held to be valid which interferes w/the general right of an individual to be free in his person and in his power to Contract in relation to his own labor. The law is no good. Reasoning: The statute necessarily interferes w/the right of Contract b/w employers and employees. The general right to make a K in relation to business is part of the liberty of the individual protected by the 14th Am. This places a limit on the police power of the state. This is not a question of substituting the judgment of the court for that of the legislature. If the act is within the power of the state, it is valid (bright line: within or not within power). Here, there is only a remote relationship b/w the act and its goal - there is no reasonable ground for interfering w/ the right of Contract by determining a bakers hours of labor. Bakers have the capacity to assert their rights and care for themselves w/o the protecting arm of the state. This law involves neither the safety, the morals nor the welfare of the public, and the interest of the public is not in the slightest degree affected by the act. There must be more than the mere fact of the possible existence of some small amount of unhealthiness to warrant interference w/liberty. (Though, some health regulations might be OK.) Slippery slope: if this statute is valid, there is no limit to which legislation of this nature might not go. Dissents: (Harlan) Whether or not this legislation is wise is not the Courts decision. Its impossible here to say there is no real or substantial relation b/w the means employed by the state and the end sought to be accomplished (the protection of health.) It should be enough to say that w/in these facts there is room for debate and for an honest difference of opinion. (Holmes) Ideological Decision - A constitution is not intended to embody a particular economic theory, whether of paternalism or of laissez faire. It is made for people of differing views. The word liberty in the 14th Am is perverted when it is held to prevent the natural outcome of a dominant opinion, unless it can be said that a rational and fair man necessarily would admit that the statute proposed would infringe fundamental principles as they have been understood by tradition. 48

Criticisms of Lochner Had no firm textual basis; DP Clause should not have been interpreted to give substantive protection Definition of individual rights to protect was either too broad or too biased toward economic liberty and property rights Didnt like the restriction of government goals Courts consideration of the common law regime as a given and criticism of legislative departures from that baseline worked a bias in favor of the status quo Classification of causal relationships b/w direct and indirect not possible The whole enterprise of the eras law was too ideological, reading one theory (laissez faire) into the Constitution, engaging in politics (Holmes in Lochner) The Courts domination was in effect a minority takeover of government (legislating from the bench)

POST-LOCHNER: 1937-PRESENT
West Coast Hotel Co. v. Parrish (1937) pg. 625 Facts: A state law established a minimum wage for women. Holding: The law is valid. Its reasonable in relation to its subject and adopted for legitimate end. Reasoning: What is this freedom of K? cuts out the heart of the Lochner era. The Const. does not speak of freedom of Contract. It speaks of liberty and prohibits the deprivation of liberty w/o due process. Regulation that is reasonable in relation to its subject and is adopted in the interests of the community is due process. Protection of women is a legitimate end of the exercise of state power. Fixing a minimum wage is an admissible means to that end. The Court cant overturn a law just b/c think its unwise. Discussion: The govt is not limited to the public interest; can engage in redistribution. Court gives deference to legislature. No longer using the language of fear. U.S. v. Carolene Products Co. (1938) pg. 626 Facts: Based on Congressional findings, Congress enacted the Filled Milk Act of 1923, which declared that filled milk is an adulterated article of food, injurious to the public health, & that its sale constitutes a fraud upon the public. It prohibited any person to ship filled milk in interstate commerce. Holding: Act is valid. Regulatory legislation affecting ordinary commercial transactions is constitutional unless it is of such a character as to preclude the assumption that it rests on a rational basis. Reasoning: The Court has to examine the facts of the situation in order to determine if there is a rational basis. In this case, the question is at least debatable whether commerce in filled milk should be left unregulated, or in some measure restricted, or wholly prohibited. That decision was for Congress. Discussion: Test is lowered to rational basis for the regulation. Almost anything is debatable, so its hard to imagine what law would not be okay under this test. Footnote 4 (pg. 627) Although the court is going to analyze constitutional challenges very leniently; the court will not do this with regards to all laws, some laws may be reviewed with strict scrutiny under the prohibitions of the 14th Amendment laws that are going to limit the democratic process which can ordinarily be expected to bring about the repeal of undesirable legislation. Williamson v. Lee Optical (1955) pg. 629 Facts: An Oklahoma statute made it unlawful for an optician to fit or duplicate lenses w/o a prescription from an ophthalmologist or optometrist. 49

Holding: The law is valid under the rational basis test. Reasoning: The law may exact a needless, wasteful requirement in many cases. But it is for the legislature, not the courts, to balance the advantages and disadvantages of the requirement. The law need not be in every respect logically consistent w/its aims to be constitutional. It is enough that there is an evil at hand for correction, and that it might be thought that the particular legislative measure was a rational way to correct it. The day is gone when the Court uses the DP Clause to strike down state laws, regulatory of business/industrial conditions, because they may be unwise, improvident, or out of harmony with a particular school of [thought]. Discussion: Rational basis is becoming the language of due process. Whatever the legislature does is pretty much considered to be in the public interest.

RATIONAL BASIS REVIEW: Economic Liberties Regulatory legislation affecting ordinary commercial transactions is constitutional UNLESS it is of such a character as to preclude the assumption that it rests on a rational basis. If regulation is rationally related to achieving a legitimate state purpose then it is constitutional. Court gives huge deference to legislative judgment.

50

FUNDAMENTAL RIGHTS UNDER DUE PROCESS & EQUAL PROTECTION


Liberalism theory of protecting fundamental rights from the democratic majority.

FUNDAMENTAL RIGHT ANALYSIS


Analysis: 1. Is there a fundamental right? a. No Fundamental Right Rational Basis Test If a challenged state action does not implicate a fundamental right, it will be upheld as reasonable relation to a legitimate state interest. b. Fundamental Right Strict Scrutiny Substantive due process forbids the government to infringe certain fundamental rights unless the infringement is narrowly tailored to serve a compelling state interest. i. Look to history and tradition (starting point). Moore v. City of East Cleveland, Ohio ii. Laws and traditions in the past 50 years are the most relevant (emerging awareness). Lawrence v. Texas 2. Is the Constitutional right infringed? a. Substantial and Direct interference? Zablocki v. Redhail 3. Is there a sufficient justification for the governments infringement of a right? a. Fundamental must show a compelling state interest b. Not Fundamental only have to show a legitimate state interest 4. Are the means sufficiently related to the ends? a. Fundamental Govt could not attain the goal through any less restrictive means. b. Not Fundamental Means are a reasonable way to achieve the goal and are not required to be the least restrictive alternative. Little depends on whether a fundamental right is found under Due Process or Equal Protection because the claimed right must be regarded as sufficiently important to be deemed fundamental even though it is not mentioned in the text of the Const. and strict scrutiny is applied to fundamental rights under both DPC & EPC. See United States v. Carolene Products Co.
Originalists Fundamental rights are limited to those explicitly stated in the text or were intended by the framers. Non-originalists Court can protect fundamental rights not enumerated in the text of the Constitution or intended by the framers. Fundamental Rights Right to Marry Loving v. Virginia, Zablocki v. Redhail Right to Procreate Skinner v. Oklahoma Right to purchase and use contraception Griswold v. Connecticut, Eisenstadt v. Baird Right to Custody of ones own child Stanley v. Illinois, Michael H. v. Gerald D. Right to Raise ones child has s/he sees fit Meyer v. Nebraska Right to Refuse medical treatment Cruzan v. Director, Missouri Dept. of Health Right to Travel freely among the states Saenz v. Roe Right to Abortion Roe v. Wade, Planned Parenthood v. Casey?

CONSTITUTIONAL PROTECTION FOR FAMILY AUTONOMY


Loving v. Virginia (1967) pg. 949 Facts: A Virginia statute made it a felony for any white person to intermarry with a colored person, or any colored person to intermarry with a white person. The Lovings were convicted. In defense

51

of the law, the state argues that the law applies equally in the sense that members of each race are punished to the same degree. Holding: Restricting the freedom to marry solely because of racial classifications violates the Equal Protection Clause. Standard of Review: The EPC requires considering whether the classifications are arbitrary and invidious discriminations. It demands that racial classifications in particular be subjected to the most rigid scrutiny (Korematsu). If they are to be upheld, they must be shown necessary to the accomplishment of some permissible state objective, independent of the discrimination itself. Reasoning: The mere equal application of a statute containing racial classifications is not enough to remove the classifications from the 14th Ams proscription of all invidious racial discriminations. This statute rests solely on distinctions drawn according to race. There is patently no legitimate overriding purpose to it independent of the racial discrimination. Concurrence: (Stewart) It is simply not possible for a state law to be valid under the Constitution, which makes the criminality of an act depend upon the race of the actor. Discussion: One of the first cases applying a different standard of review to a racial classification. The Court waited many years after Brown to get to this issue, for fear of outpacing general public sentiment. By this point Congress had jumped onto the bandwagon, passing the Civil Rights Act of 1964/65. Commentary: Answer to what makes a right fundamental will effect the application of the fundamental right. Ex. If marriage is fundamental because of procreation, then it is not fundamental for same sex marriage. Zablocki v. Redhail (1978) pg. 950 Under Rational Basis test statute would be clearly upheld. Statute violates EPC by discriminating against a specified group of people. Also violates DPC right to marry. There is a compelling state interest (making sure children are provided for), but statute is not narrowly tailored. Fails Strict Scrutiny Test. Degree of infringement determines standard of review. Strict scrutiny must be a direct & substantial interference. Concurrence: (Stewart) EPC deals only with discriminatory classifications. Problem in this case is not one of discriminatory classifications but with unwarranted encroachment of Due Process. Proposition 8 Review Courts never declare whether gay marriage is fundamental because the ban failed rational basis review. Stanley v. Illinois (1972) pg. 956 Found fundamental right of custody for biological father (DPC). Statute presuming fathers of illegitimate children unfit for custody violated EPC by discriminating against a group of persons (unwed fathers). The Court acknowledges that the State has a legitimate interest in prompt efficacious procedures furthered by the presumption that unwed fathers are unfit. However, such an interest is outweighed by the fathers right to demonstrate that he is fit when the right taken away from the father would be the right to raise his child (fundamental right). Michael H. v. Gerald D. (1989) pg. 958 Facts: A California statute provided that a child born to a married woman living with her husband is conclusively presumed to be a child of the marriage. This presumption has consequences for the visitation rights of the genetic father of such a child. Holding: The statute is valid. The liberty interest at stake here is not fundamental. 52

Reasoning: The state interest is protecting the marital union of their citizens. Michaels parental relationship adulterously conceived is not protected by tradition. Michael must show that that society has traditionally accorded the father of an illegitimate child, conceived adulterously, parental rights, or at least has not traditionally denied them. Rationale of precedent was in protecting the family unit, not just one member of the unit. Dissent: (Brennan) The interest here is that of a parent and child in their relationship with one another. If test is to say whether it is protected by history the Due Process Clause is redundant. Would contradict precedent. Discussion: Scalia adds a footnote discussing what level of generality is appropriate when formulating the interest at stake. He argues that it should be the most specific level at which a relevant tradition protecting, or denying protection to, the asserted right can be identified. OConnor & Kennedy This footnote sketches a mode of historical analysis that maybe somewhat inconsistent with our past decisions in this area. Brennan says the Court has not used such specificity before and requiring it will squash our liberties. *This case returned to using deeply rooted traditions rather than emerging consensus, because it wanted to get back in sync with popular opinion. (Interest must be fundamental but also rooted in American tradition). Distinguished from Stanley because Due Process protects the family unit not the individual Belle Terre v. Boraas (1974) pg. 968 [The case] involved no fundamental right guaranteed by the Constitution. Rational Basis review. Moore v. City of East Cleveland, Ohio (1977) pg. 964 Plurality Facts: A city ordinance limited occupancy of any dwelling unit to members of the same family, where the ordinance narrowly defined family as including only a few categories of related individuals. Moore lived with her son and her two grandsons. Under the ordinance, one of the grandsons could not live in the home b/c he was insufficiently related to his uncle and his cousin to constitute a family. Holding: The ordinance violates the DPC. The goal is legitimate, but the fit of the means is marginal at best. Standard of Review: when govt intrudes on choices concerning family living arrangements, the Court must examine the importance of the govt interest and the extent to which the regulation serves them. Reasoning: Freedom of personal choice in matters of marriage and family life has long been recognized by this Court as one of the liberties protected by the DPC. Institution of family is deeply rooted in this Nations History and Tradition. Extended family also rooted in history and tradition and is protected. This is unlike the ordinance we upheld in Belle Terre b/c that one affected only unrelated individuals (implying there is NO liberty right to live together except in blood relationships). FEAR: return to the Lochner era of judicial intervention history counsels caution and restraint, but not abandonment. We need limits on substantive DP, but the family is definitely protected. Dissent: (Stewart) To suggest that related people have const rights of assoc superior to those of unrelated people is to misunderstand the nature of freedom of association. Freedom of association is protected b/c it is considered indispensable to effectuation of 1st Am guarantees. The association in this case is not for any purpose relating to the 1st Am.

53

Meyer v. Nebraska (1923) pg. 968 Rule of Law: The Fourteenth Amendment prohibits states from creating legislation that restricts liberty interests when the legislation is not reasonably related (Lochner) to an acceptable state objective. Facts: Plaintiff was convicted for teaching a child German under a Nebraska statute that outlawed the teaching of foreign languages to students that had not yet completed the eighth grade. The Supreme Court of Nebraska upheld the conviction. Issue: Does the statute as construed and applied unreasonably infringe on the liberty guaranteed by the Fourteenth Amendment? Holding: There is a legitimate state interest. The statute as-applied is unconstitutional because it infringes on the liberty interests of the plaintiff and fails to reasonably relate to any end within the competency of the state. The Fourteenth Amendment encompasses more than merely the freedom from bodily restraint. The state argues that the purpose of the statute is to encourage the English language to be the native tongue of all children raised in the state. Nonetheless, the protection of the Constitution extends to those who speak other languages. Education is a fundamental liberty interest that must be protected, and mere knowledge of the German language cannot be reasonably regarded as harmful. Discussion: Liberty interests may not be interfered with by the states when the interference is arbitrary and not reasonably related to a purpose, which the state may permissively regulate.

CONTRACEPTION CASES
Griswold v. Connecticut (1965) pg. 980 Facts: A CT statute prohibits any person to use any drug, medicinal article or instrument for the purpose of preventing conception. Punishable by one year in prison (aiding and abetting) Appellants are Griswold, Executive Director of the Planned Parenthood League of Connecticut, and Buxton, licensed physician and professor at Yale Med who served as Medical Director for the League. They gave info, instruction, and medical advice to married persons as to the means of preventing contraception. Fees were usually charged, although some couples were serviced free. They were fined under the CT statute. Holding: The law must be struck down. Right to Privacy under Bill of Rights. Standard of Review: A govt purpose to control or prevent activities constitutionally subject to state regulation may not be achieved by means, which sweep unnecessarily broadly and thereby invade the area of protected freedoms. Reasoning: The Court cannot sit as a super-legislature determining the wisdom of various laws. But the Court does have to protect rights, including peripheral rights. We have found other peripheral rights before (e.g., right to free association). In other contexts weve found that the 1st Am has a penumbra where privacy is protected from govt intrusion. We believe specific guarantees in the Bill of Rights have penumbras formed by emanations from those guarantees that help give them life and substance. Various guarantees create zones of privacy. This case concerns a relationship lying w/in the zone of privacy created by several fundamental Const. guarantees. The law, in forbidding the use of contraceptives rather than their manufacture/sale, seeks to achieve its goals by means having a maximum destructive impact. In fact, it could not be enforced we would not let police search marital bedrooms for telltale signs of violation. The right of privacy in marriage is older than the Bill of Rights. Distinction from Lochner is economic vs. social regulations. Concurrence: (Goldberg) Right to Privacy under the 9th Amendment. The conclusion that the concept of liberty embraces the right of marital privacy though that right is not mentioned explicitly in the Const. is supported by the 9th Am, which reveals that the framers believed that there are additional fundamental rights protected from govt infringement. Were not applying the 9th Am 54

against the states, just using it as evidence of intent. *To tell which rights are fundamental, we must look to the traditions and collective conscience of our people. The inquiry is whether the right involved can be denied w/o violating fundamental principles of liberty and justice which lie at the base all our civil and political institutions.* (Harlan) Right to Privacy under Due Process Clause of 14th Amendment. We can decide this case just based on the DPC. The principle of marital privacy is not in the Const text, but is a fundamental principle. Certain interests require particularly careful scrutiny of the state needs asserted to justify their abridgement. The choice of means also matters. This statute should get strict scrutiny. (White) The statute does not in any way further the govt purpose of preventing illicit sexual relationships. Dissent: (Black) There is no right to privacy in the Const. The govt has a right to invade privacy unless prohibited by some specific Const provision. The 9th Am is just supposed to limit the fed govt to the powers granted expressly or by necessary implication. (Stewart) This is a silly law, but that isnt our call. Discussion: Palko/Adamson approach: focusing on interests whose protection is rooted deeply in our traditions. This case is trying to limit the scope of that to marriage/the home. 9th Amendment The enumeration in the Constitution, of certain rights, shall not be construed to deny or disparage others retained by the people. Not a source of rights only a textual justification for the Court to protect non-textual rights. Conservatives ignore amendment (like an ink blot on the Constitution in which justices cannot interpret). Amendment gives great discretion to the Court in determination of fundamental rights (too much for Conservatives). Eisenstadt v. Baird (1972) pg. 986 Facts: A Mass. statute prohibited the distribution of any drug or device to unmarried persons for the prevention of contraception. Holding: The statute violates the EPC b/c it provides dissimilar treatment for married and unmarried persons. Standard of Review: Rational basis (but not applied as such) Reasoning: The govt has asserted no sufficient interest. The effect of the law (means) had at best a marginal relation to the proffered objective of deterring premarital sex (State interest); and fornication is already a misdemeanor, so the state cant possibly be prescribing pregnancy as punishment for that same crime. Its true that in Griswold the right of privacy in question inhered in the marital relationship. Yet the marital couple is not an independent entity w/a mind and heart of its own, but an association of two individuals. If the right of privacy means anything, it is the right of the individual, married or single, to be free from unwarranted govt intrusion into matters so fundamentally affecting a person as the decision whether to bear or beget a child. Right to Privacy protects an individuals right to bare a child. Essentially decided on Equal Protection Clause, but addresses Due Process in Dicta.

ABORTION CASES
Roe v. Wade (1973) pg. 989 Facts: Texas statutes make procuring an abortion a crime except by medical advice for the purpose of saving the life of the mother. Statutes like them have been in effect in many states for about a century. Holding: States cannot regulate the first trimester at all; b/w first trimester and viability, can regulate only for health reasons; after viability, can regulate/proscribe abortion except where medically necessary for the preservation of the life or health of the mother. 55

Standard of Review: Strict scrutiny: Where certain fundamental rights are involved, regulation limiting these rights may be justified only by a compelling state interest, and it must be narrowly drawn to express only the legitimate state interests at stake. Reasoning: We acknowledge the controversy around this but try to resolve it as a legal matter. We note that abortion was allowed at the time of the framing more than it is today. Texas says life begins at conception. Court says that the word person is not used in the 14th Am does not include the unborn: Justify by saying first, abortion not regulated until the late 19th century, and second, that they have the power to define person. - Fundamental right issue: The Const does NOT Explicitly mention any right of privacy, but the Court has recognized that a right of personal privacy, or a guarantee of certain areas or zones of privacy, does exist under the Const (Griswold). Only fundamental rights (implicit in the concept of ordered liberty) are included in this guarantee of personal privacy. Also, its clear the right has some extension to activities relating to marriage (Loving), or contraception (Eisenstadt), etc. This right of privacy (in the DPCs liberty concept) is broad enough to encompass the right to decide to have an abortion. - Compelling state interest issue: The state has important interest in protecting the health of the pregnant mother. The state also has an interest in protecting life, but a fetus is not a person so that doesnt apply. However, there is an interest in protecting potential life. These interests are separate and distinct. Each grows as the woman reaches term and, at some point during the pregnancy, becomes compelling. For the interest in potential health, that point becomes compelling around the end of the first trimester (b/c before that point, mortality from abortion is less than mortality in normal childbirth). For the interest in protecting potential life, the compelling point is at viability (b/c the fetus then has the capability of meaningful life outside the womb). After each compelling point begins, the state may regulate THAT PARTICULAR INTEREST.
Rule pg. 995

Concurrence: (Stewart) The Const does not mention a specific right of personal choice in matters of marriage and family life, but the liberty protected by the DPC covers more than those freedoms explicitly named in the Bill of Rights. The right listed in Eisenstadt covers abortions. The Texas statute infringes that right directly. Dissent: (White) Theres no textual basis for this judgment. The Court should have left this to the political process. (Rehnquist) An abortion is not private. The Courts weighing of competing factors (substituting for the established test) is more appropriate for legislative judgment. The right to have an abortion is NOT fundamental. The fact that states have had prohibitions against it for a century shows that it is not rooted in the traditions and conscience of our people. Liberties under the 14th Amendment are not guaranteed absolutely against deprivation, only against deprivation without due process of law. Discussion: The measurement metaphor recalls Lochner. In fact, the Court here was reviving a mode of argument central in the Lochner period. It is the first case since Lochner to look at liberty in the 14th Am to create a substantive right to privacy of decision-making. Also uses Palko/Adamson approach: idea that word liberty in 14th Am provides a way to protect values essential to liberty. Can show consensus rooted in tradition (Americans decide value is essential); or can show trend of contemporary opinion (in statutes, public debate, etc.) It was fairly easy to find a consensus in Griswold and Eisenstadt. Here, not so much. Blackmun turned this tethering upside down by pointing to the distant past; he doesnt tether but rejects public opinion instead. He could have pointed to a trend emerging to deregulate abortion, but in 1973 it was just beginning, and the Court didnt want to wait for the parade. He could NOT make a Carolene footnote 4 argument b/c in 1973 the Court refused to declare gender a suspect classification (Frontiero) not a discrete/insular group paralyzed by prejudice. 56

Planned Parenthood of Southeastern Pennsylvania v. Casey (1992) pg. 998 Facts: PA Abortion Control Act of 1982 requires that a woman seeking an abortion give her informed consent prior to the abortion procedure, and specifies that she be provided w/ certain info at least 24 hours before the abortion is performed. For a minor to obtain an abortion, the Act requires the informed consent of one of her parents, but provides for a judicial bypass option if the minor does not wish to or cannot obtain a parents consent. Another provision requires that, barring some exceptions, a married woman seeking an abortion must sign a statement indicating that she has notified her husband of her intended abortion. The Act exempts these three requirements in the event of a medical emergency. Holding: Reaffirms Roe holding: Recognizes right of woman to choose to have an abortion before viability and to obtain it w/o undue burden/interference from the state; Confirms states power to restrict abortions after viability if the law contains exceptions for health endangerment; Reaffirms states legitimate interests from the outset in protecting the womans health and the potential life of the fetus (BUT rejects trimester framework). Standard of Review: A regulation is an undue burden when it has the purpose or effect of placing a substantial obstacle in the path of a woman seeking an abortion of a nonviable fetus. Reasoning: The DPC contains a substantive component. It is not limited by the Bill of Rights or the specific practices of states at the time of adoption of the 14th Am. - Stare Decisis Analysis: Has it become unworkable? No. Has there been reliance on it? Yes. Has legal principle evolved so as to leave its doctrinal footings weaker? No. The Court then compares Roe to other cases that have been controversial in the past that were later overturned. Lochner: a change in facts justified overruling it (the Depression showed that an unregulated market wasnt sufficient to sustain human welfare). Here, there has been no change in facts/circumstances. We cant just overrule a case because we think it was wrong. FEAR: undermining Courts credibility. - As applied to this statute: The information requirement is valid as long as the info is truthful and not misleading. Giving her the info is not a substantial obstacle. The waiting period aspect of that requirement is also okay because it is an obstacle, but not too substantial it is only a persuasive measure. The husband notification requirement IS an undue burden b/c of the possibility of abusive husbands. The husband has an interest in the life of the child, but that does not empower him with a troubling degree of authority over his wife (different than the dominion exercised by parents over children). The parental consent requirement is okay. Concurrence/Dissents: (Blackmun) Mentions the right-to-bodily-integrity argument and the decisional-autonomy argument. He would hold the 24-hour waiting period an undue burden. (Rehnquist) Roe was wrong and can be overturned consistent with stare decisis. This whole statute should be upheld. A womans decision to abort is not a fundamental right requiring strict scrutiny. This statute is rationally related to a legitimate state interest. (Scalia) Abortion is not protected by the Const b/c (1) the text says nothing about it, and (2) the longstanding traditions of American society have permitted it to be regulated/banned. Under rational basis test, the PA statute should be upheld. We should leave these issues to the political process. Rejects trimester framework from Roe. Adopts the Undue Burden Standard pg. 1004 applies only to abortion cases. Gonzales v. Carhart (2007) pg. 1010 Facts: Partial Birth Abortion Ban Act of 2003 makes it a crime to perform an intact D&E, essentially the same procedure as in Stenberg but with a few changes. The PBA was a response to the decision in Stenberg, and prohibits knowingly performing a birth and then killing the partially delivered fetus. It did not prohibit D&E where the fetus is dismembered inside the body. It did not contain an exception for mothers health. Congress as a fact-finder said that the PBA is never medically 57

necessary. The plaintiff is challenging this as facially unconstitutional, not applied. Undue Burden Standard: If the act subjects women to significant health risks then it is unconstitutional. It does not. Holding (Kennedy): o This is not unconstitutional on its face. o It is not vague, it gives clearly anatomical landmarks to indicate division between forbidden abortion methods and permitted ones. o It does not bar D&E, which was a concern in Stenberg. o It is not an undue burden: Congress is free to legislate to show profound respect for human life and further its legitimate interest in regulating profession to promote respect for life. Protection of a mothers health Some when come to regret their choice. Doctors dont always tell women the precise details of how theyre going to effect the abortion. So they struggle with grief when they know. The medical profession can find other ways of doing this. This may encourage some women to carry infant to full term and reduce total number of late-term abortions. Lack of exception for womans health is not an undue burden. There is documented medical disagreement whether acts prohibition ever imposes significant health risks. Legislature has wide discretion to legislate where there is scientific uncertainty. o Court is only deciding this as a broad facial challenge. Plaintiffs could bring an as-applied challenge if the lack of health exception really meant mothers health would be placed in jeopardy. Concurrence (Thomas, Scalia): This accurately applies the current jurisprudence; but abortion jurisprudence has no basis in the Constitution. Dissent (Ginsberg): o Wants an exception for the health of the mother. This is a safer procedure for women. o This procedure doesnt save a single fetus, since another procedure can always be used. o Doesnt like regret rationale. Women are required to be informed. You are depriving them of the right to make autonomous choices. This way of thinking reflects ancient notions about womens place. o Court is chipping away at a right declared by this Court. WHAT DOES GONZALES MEAN TODAY? o Government can sometimes completely ban a particular method so long as it is not an undue burden, which apparently means there is another reasonably safe method (even if there is scientific uncertainty) on that point. o Government can take into account perceived risks to mental health of women including regret. This goes to consent provisions probably. o Legislatures findings as to safety and scientific uncertainty prevail. Even if doctor thinks procedure is safer for mother, no exemption is required where legislature has made credible findings. State Interests in Abortion Regulation: Health of the mother, protecting prenatal life (not necessarily served). Moral concerns also an interest in this case (respect & dignity for human life), advancement of the medical profession. Facial challenge vs. As Applied challenge.

58

ABORTION RESTRICTIONS (5)


1. Waiting Period Rule a state is permitted to enact persuasive measures which favor childbirth over abortion, even if those measures do not further a health interest. 24-hour waiting period 2. Government Restrictions on Funding Government do not have to provide funding for nontherapeutic abortions when it provides funding for childbirth. Also not required to provide funding for indigent women except in life threatening circumstances. Maher v. Roe (1977) pg. 1026 Facts: A state regulation granted Medicaid benefits for childbirth but denied benefits for nontherapeutic abortions. Holding: The statute does not violate DPC - does not infringe on the fundamental right recognized in Roe. Standard of Review: Rational Basis Reasoning: This case involves no discrimination against a suspect class. Women who desire an abortion are not one; and the Court has never held that financial need alone identifies a suspect class. Does the regulation infringe on a fundamental right? No. Roe did not declare an unqualified constitutional right to abortion. The right only protects the woman from unduly burdensome interference w/her freedom to decide to have an abortion. It does not limit the authority of a state to make a value judgment favoring childbirth over abortion and implementing that judgment by allocating public funds accordingly. The regulation places no obstacles (absolute or otherwise) on abortions. No restrictions on access have been imposed; childbirth just may be made a more attractive alternative. Theres a distinction b/w prohibition of one activity and encouragement of another activity. SO because theres no fundamental right being infringed, rational basis review is applied. The state unquestionably has a strong interest in encouraging normal childbirth. The regulation unquestionably rationally furthers that interest. Dissent: (Brennan) The regulation impinges on the right of privacy by bringing financial pressures on indigent women that force them to bear children they would not otherwise have. Its not an absolute bar, but it is unduly burdensome. (Marshall) The regulation is just trying to impose a moral viewpoint. Harris v. McRae (1980) pg. 1028 Facts: The Hyde Amendment prohibited the use of federal Medicaid funds to perform abortions except where the life of the mother would be endangered if the fetus were carried to term; or except for such medical procedures necessary for the victims of rape or incest. Holding: The law is upheld. Standard of Review: Rational Basis Reasoning: A womans interest in protecting her health was an important theme in Roe, but it does not follow that a womans freedom of choice carries with it a constitutional entitlement to the financial resources to avail herself of the full range of protected choices. Govt may not place obstacles in the path of a womans exercise of free choice, but it need not remove those not of its own creation (i.e., indigency). Congress hasnt invaded a fundamental right, not legislated against a suspect class. Thus, only rational basis review and its not irrational for Congress to authorize federal reimbursement for medically necessary services generally but not for medically necessary abortions, b/c no other medical procedure involves purposeful termination of potential life. Dissent: (Brennan) The state cant unduly burden the right to abortion. (Stevens) State interference is unreasonable if it attaches a greater importance to the interest of the potential life than to the interest in protecting the mothers health.

59

3. Spousal Consent requirement for abortions is unconstitutional. Planned Parenthood v. Danforth (1976) pg. 1030 4. Spousal Notification requirement for abortions is unconstitutional. Planned Parenthood of Southeastern Pennsylvania v. Casey (1992) pg. 1031 5. Parental Notice & Consent state can require parental notice or consent as long as it includes a judicial waiver alternative.

CONSTITUTIONAL PROTECTION FOR MEDICAL DECISIONS


Cruzan v. Director, Missouri Dept. of Health (1990) pg. 1041 Facts: Cruzan had a car accident. She was in a coma and then progressed to an unconscious state in which she was able to orally ingest some nutrition. She had to have a feeding/hydration tube inserted, w/the consent of her then husband. Shes now in a persistent vegetative state. The state of Missouri is bearing the cost of her care. Its become apparent she has virtually no chance of regaining her mental facilities. Her parents asked the hospital to take out the feeding tube, and they said no w/o court approval. Holding: The state has the right to require proof of clear and convincing evidence of a wish to have treatment withdrawn. Standard of Review: seems to be applying strict scrutiny? Reasoning: Theres a principle that a competent person has a constitutionally protected liberty interest in refusing unwanted medical treatment. We assume that the Const would also grant a competent person a protected right to refuse life-saving nutrition and hydration. But an incompetent person is not able to make an informed and voluntary choice to exercise this hypothetical right, or any other right. The right can be exercised by a surrogate, but the state requires that it be done only with clear and convincing evidence that thats the incompetents wish. The Const does not forbid this procedural requirement. The state has a very strong interest in the protection and preservation of human life. It may also seek to safeguard the personal element of the choice through heightened evidentiary requirements. Based on the evidence in this case, the state made a valid conclusion. Concurrence: (OConnor) The Court has often deemed state incursions into the body repugnant to the interests protected by the DPC. (Scalia) Federal courts have no business in this field. American law has always accorded the state the power to prevent suicide, by force if necessary. No substantive due process claim can be maintained unless the P demonstrates that the state has deprived him of a fundamental right (a right historically and traditionally protected against state interference) that cannot be established here. This Court need not, and has no authority to, inject itself into every field of human activity where irrationality and oppression may theoretically occur, and if it tries to do so it will destroy itself. Dissent: (Brennan) No state interest could outweigh the rights of someone in Cruzans position. Until her wishes have been determined, the only state interest that may be asserted is the interest in safeguarding the accuracy of that determination. This rule imposes an asymmetrical evidentiary burden. Suggests a preponderance of the evidence burden of proof because it will be more accurate in the situation where it is inappropriate to err on either side of caution. (Stevens) This requirement doesnt pass rational basis review. The end is not legitimate: it is just an effort to define life. There is no ground for believing that Cruzan has any personal interest in the perpetuation of what the state has decided is her life. Discussion: This came before Casey or Lawrence. No fundamental right to decide to kill yourself. Less application today because most people have a living will.

60

Analysis: 1. Is there a liberty interest? 2. Determination of a violation of a liberty interest must be balanced against relevant state interest. Missouri Interest protection and preservation of human life Cruzan Interest Protection from battery (refusal of unwanted medical treatment) Washington v. Glucksberg (1997) pg. 1048 Facts: A Washington law provides: A person is guilty of promoting a suicide attempt when he knowingly causes or aids another person to attempt suicide. Washingtons Natural Death Act states that the withholding or withdrawal of life-sustaining treatment at a patients direction shall not, for any purpose, constitute a suicide. As-Applied Challenge. Holding: The prohibition on suicide is valid. Theres no fundamental right to kill yourself. Standard of Review: The DPC specially protects fundamental rights and liberties deeply rooted in history and tradition and implicit in the concept of ordered liberty (Palko); the liberty interest must be carefully described. If no fundamental right, only Rational Basis review. Reasoning: In almost every state, and every western country, its a crime to commit suicide. Theres a long tradition of preserving human life. The suicide bans have been reaffirmed in recent years. We require a careful description of the liberty interest to rein in the subjective elements necessarily present in DPC review. The fundamental right requirement avoids the need for complex balancing of competing interests in every case. There is not one here. Its different from Cruzan, which involved refusing medical treatment, not suicide; that has a stronger basis in the common law tradition. Also, Casey isnt controlling. Just b/c many of the rights protected by the DPC sound in personal autonomy does not warrant the sweeping conclusion that ALL intimate and personal decisions are so protected. No fundamental liberty here rational basis review. The states interest here is strong: the preservation of human life. The prohibition on suicide rationally advances this interest. Also an interest in protecting the integrity and ethics of the medical profession. Slippery slope: towards involuntary euthanasia? Concurrence: (OConnor) There is no generalized right to commit suicide. (Stevens) Just b/c the statute is valid on its face doesnt mean it wont be invalid as applied. (Souter) Lets wait and see what happens in the Netherlands; theyve got some experience with physician-assisted suicide and euthanasia. Doctrine of Double Affect doing a deed for the right reason, but achieving an incidental evil, is still okay. A doctor attempting to alleviate pain and suffering with medication that is going to hasten death as a result cannot be criminalized conduct. Discussion: This case came between Casey and Lawrence. Who is the Court considering when its trying to tether? Probably all Americans. Should it be the specific group affected? (No, probably) SCOTUS Manipulation of the framing of fundamental rights in order to fix the analysis in their favor. Vacco v. Quill (1997) pg. 1054 Facts: Dr. Timothy E. Quill, along with other physicians and three seriously ill patients who have since died, challenged the constitutionality of the New York State's ban on physician-assisted suicide. New York's ban, while permitting patients to refuse lifesaving treatment on their own, has historically made it a crime for doctors to help patients commit or attempt suicide, even if patients are terminally ill or in great pain. Question: Did New York's ban on physician-assisted suicide violate the Fourteenth Amendment's Equal Protection Clause by allowing competent terminally ill adults to withdraw their own lifesaving treatment to hasten death, but denying the same right to terminally ill patients who are not on life 61

support and could only hope that a physician would do so for them? Standard of Review: Rational Basis Equal Protection Holding: No. Employing a rationality test to examine the guarantees of the Equal Protection Clause, the Court held that New York's ban was rationally related to the state's legitimate interest in protecting medical ethics, preventing euthanasia, shielding the disabled and terminally ill from prejudice which might encourage them to end their lives, and, above all, the preservation of human life. Moreover, while acknowledging the difficulty of its task, the Court distinguished between the refusal of lifesaving treatment and assisted suicide, by noting that the latter involves the criminal elements of causation and intent. No matter how noble a physician's motives may be, he may not deliberately cause, hasten, or aid a patient's death. Distinction between withdrawing life sustaining medical treatment & assisted suicide A doctor who withdraws life sustaining treatment only intends to respect the wishes of his patient; a doctor who assist in suicide must necessarily and indubitably, intend primarily that the patient be made dead.

CONSTITUTIONAL PROTECTION FOR SEXUAL ORIENTATION & SEXUAL ACTIVITY


Bowers v. Hardwick (1986) pg. 1056 Facts: Georgia had a sodomy statute defining sodomy as committing or submitting to any sexual act involving the sex organs of one person and the mouth or anus of another. Plaintiff is an adult male criminally charged for violating the statute with another male in his own bedroom. Holding: The statute is upheld. Standard of Review: Only fundamental liberties implicit in the concept of ordered liberty (Palko), deeply rooted in this nations history and tradition (Griswold), get heightened judicial protection. Rational Basis Review. Reasoning: There is no fundamental right of homosexuals to engage in acts of consensual sodomy. None of the rights announced in Pierce, Skinner, Griswold, Roe bears any resemblance to this claimed right. There is no connection b/w family/marriage/procreation and homosexual activity. There is no right to private sexual conduct b/w consenting adults, either. Proscriptions against sodomy have ancient roots, so you cant claim that the right is rooted in tradition or implicit in our concept of liberty. FEAR: the Courts own vulnerability and illegitimacy the Court opens itself up to risk when it deals with judge-made Const. law not in the text. Were going to stay out of this. Slippery slope: expanding this right of privacy could create all sorts of rights that we do not want. Its okay for a law to be based on notions of morality. If all laws based on moral choices were invalidated under the DPC, wed have none left. Discussion: Opinion is framed as homosexual sodomy as a fundamental right even though statute implicates heterosexual sodomy as well. Rational Basis Review Legitimate state interest is morality. The court believes morality is sufficient as a legitimate state interest. Lawrence v. Texas (2003) pg. 1056 Overrules Bowers Review this Case Facts: A Texas statute makes it a criminal act to engage in deviate sexual intercourse with another individual of the same sex. Deviate sexual intercourse is any contact b/w any part of the genitals of one person and the mouth or anus of another person, or the penetration of the genitals or the anus of another person with an object. Lawrence was arrested for violating this statute in his own apartment. Holding: The statute is invalid; Bowers is overruled. Standard of Review: Rational Basis! (Wrong standard) Govt needs to show that statute furthers a legit state interest.

62

Reasoning: First the Court gives an overview of relevant precedent (Griswold, Eisenstadt, Roe, Carey, Bowers). The right at stake was formulated too narrowly in Bowers. Yes, the statutes prohibit a particular sexual act, but their penalties/purposes have more far-reaching consequences, touching upon private human conduct, sexual behavior, and the sanctity of the home. There is a right to enter an intimate relationship in the confine of your home and your own private life. Bowers placed too much emphasis on history. Laws against sodomy may still be on the books, but theyre never enforced anymore. AND, laws targeting same-sex couples didnt develop until about 30-40 years ago. There is now an emerging awareness that liberty protects adult persons in deciding how to conduct their private lives in matters pertaining to sex. Bowers has been eroded by Casey and Romer. Stare decisis is important, but not an inexorable command. The factors we talked about in Casey about whether to overrule Roe are different here there hasnt been reliance; the rationale of Bowers does not withstand careful analysis. It was wrong in the first place and its wrong now. The statute furthers no legit state interest; which can justify its intrusion into the personal and private life of the individual. Were using the DPC rather than the EPC so that we can get rid of all anti-sodomy laws. If we used the EPC, states would be able to keep the laws as long as they prohibited same-sex and different-sex activities equally. Concurrence: (OConnor) I dont join the Court in overruling Bowers. This statute, however, is invalid under the Equal Protection Clause. Moral disapproval of a group, like a bare desire to harm the group, is an interest that is insufficient to satisfy rational basis review under the EPC. It cannot be a legit govt interest b/c legal classifications must not be drawn for the purpose of disadvantaging the burdened group. Dissent: (Scalia) No mentioning of a fundamental right for homosexual sodomy, and no strict scrutiny review (courts tend to agree. We should be consistent when invoking stare decisis. This case is totally inconsistent with Caseys use of that doctrine. Roe is just as overrulable as Bowers under the conditions discussed here. This case will cause a massive disruption of the current social order b/c it invalidates tons of other laws based on moral choices (laws against bigamy, incest, prostitution). Homosexual sodomy is not a fundamental right. An emerging awareness cannot be enough; we also need tradition. *History & Tradition test not that homosexual sodomy was not criminalized in that time, but that the right has been respected (Raban agrees with this point). (Thomas) This case is silly, but its not our call. Discussion: Kennedy vacillates between tethering to a popular warrant (tradition, consensus) and falling into Lochner-type arguments. Many people prefer OConnors approach because majority applied rational basis test when it should have used strict scrutiny. EPC analysis is more consistent w/democratic authority it allows the govt to continue pursuing its goal just not through discriminatory classifications. However, this is exactly why Kennedy says he DOESNT want to use it. Also, its likely that using EPC would have put more pressure on them to decide the same-sex marriage issue sooner. Using DPC let them bracket the issue. Heightened Rational Basis review for statutes animus towards a certain group of people. Court never explicitly addresses. Courts can establish a principle that governs in a case but the principle cannot be made applicable to other areas unless it comes before the Court. Courts may or may not choose to apply the principle on their own. Alabama Sex Toy Case 11th Circuit validates statute prohibiting the sale of sex toys within the state, on Bowers precedent rather than Lawrence
__________________________________________________________________________________________

63

APPENDIX
SEPARATION OF POWERS
to Legislative Legislative in relation to
Delegation of Powers Bicameralism INS v. Chada Removal of Legislative officers

to Judicial
Assignment of Nonjudiciary tasks Article I Legislative Courts Privilege against Congressional Arrest Judicial Inquiry into Congressional doings Legislative re-opening of final judgments Independence, Impartiality Life Tenure

to Executive
Removal of Executive officers Presentment Executive inquiry into Congressional doings Executive Immunities & Privileges Judicial, not Executive Duties Marbury v. Madison Executive Immunities & Privileges Political Question Removal of Executive officers

Judicial in relation to

Will not re-open final judgments Political Question

Executive in relation to

Removal of Presidential Officers Congress cannot override, unless for cause Veto power Cannot inquire into Congressional doings Executive Immunities & Privileges

Executive Immunities & Privileges Political Question

LIST OF CONSTITUTIONAL CLAUSES


Article 1 - Legislative Commerce Clause 8 Clause 3 Congress has the power to regulate commerce with foreign nations, and among the several states, and with the Indian tribes. Necessary and Proper Clause 8 Clause 18 To make all laws which shall be necessary and proper for carrying into execution the foregoing powers [of Section 8] Borrow money for the United States; Regulate commerce; Declare war; Raise and support armies Suspension Clause - 9 Clause 2 The Privilege of the Writ of Habeas Corpus shall not be suspended, unless when in Cases of Rebellion or Invasion the public Safety may require it. Contracts Clause 10 No state shallpass any law impairing the Obligation of Contracts. Article 3 - Judicial Exceptions & Regulations Clause 2 In all other cases before mentioned, the Supreme Court shall have appellate jurisdiction both, as to law and fact, with such exceptions, and under such regulations as the Congress shall make. Case & Controversy Clause 2 Clause 1 *Political Question falls under separation of powers NOT case & controversy clause but SCOTUS has addressed it as a part of C&C clause.

64

Article 4 Guaranty Clause 4 The United States shall guarantee to every State in this Union a Republican Form of Government, and shall protect each of them against Invasion; and on Application of the Legislature, or of the Executive (when the Legislature cannot be convened) against domestic Violence. Article 6 Supremacy Clause Clause 2 When statutes conflict with the Constitution, the constitution governs. Amendments 13th Enabling Clause permits Congress to pass all laws necessary and proper for abolishing all badges and incidents of slavery in the United States 14th 1 o Privilege & Immunities Clause No State shall make or enforce any law, which shall abridge the privileges or immunities of citizens of the United States; o Due Process Clause nor shall any State deprive any person of life, liberty, or property, without due process of law; o Equal Protection Clause nor deny to any person within its jurisdiction the equal protection of the laws.

65

Das könnte Ihnen auch gefallen